This interplay leads to rapid electrical activity in the atria, which can supersede the SA node and produce the disorganized and ineffective atrial contractions.
The rapid atrial electrical activity is also conducted through the atrioventricular (AV) node, leading to an irregular ventricular response.
Atrial Fibrillation rarely exists in isolation and is typically a manifestation of other underlying disease processes.
Among patients with AF, 80% have cardiovascular disease, most commonly hypertension, coronary artery disease (CAD), valvular heart disease, congenital, heart failure, previous heart surgery, and cardiomyopathy.
Abnormalities or damage to the heart's structure are the most common cause of atrial fibrillation. Possible causes of atrial fibrillation include:
Increasing age,
chronic obstructive pulmonary disease [COPD]),
hyperthyroidism, sepsis, pulmonary embolism, and electrolyte abnormalities.
obstructive sleep apnea, obesity, and diabetes.
Exposure to stimulants, such as medications, caffeine, tobacco or alcohol
Sick sinus syndrome — improper functioning of the heart's natural pacemaker
Viral infections
Stress due to pneumonia, surgery or other illnesses
The exception is lone AF, which is a term used to describe AF in patients younger than In lone atrial fibrillation, the cause is often unclear, and serious complications are rare.}
+++++++
However, some people who have atrial fibrillation don't have any heart defects or damage, a condition called lone atrial fibrillation. In lone atrial fibrillation, the cause is often unclear, and serious complications are rare.
Below is a short video which will help you quickly identify atrial fibrillation on a monitor.
Please allow several seconds for the video to load.
AF is initiated by premature beats. Triggering foci are an important cause of this arrhythmia.
Persistent AF is associated with atrial structural and electrophysiologic remodeling, as well as with triggering foci.
Long-standing persistent AF is associated with greater structural remodeling with atrial fibrosis and electrophysiologic remodeling.
AF is initiated by rapid firing (or triggers) from the pulmonary veins (PV)??????.
Early in the course of AF the atrium is relatively healthy and as a result sinus rhythm is spontaneously restored.
As the substrate remodels further over time, AF no longer terminates spontaneously and becomes persistent. With more extensive remodeling of the atrium, it becomes increasingly difficult to maintain sinus rhythm and the patient and physician may agree no longer to attempt to maintain sinus rhythm, with the AF thereby being considered permanent [10].
Triggers of AF — It has been known for many years that a single focus firing rapidly in the atria can be a trigger for fibrillatory conduction throughout the atria [11]. It is now well established that the most common site of the rapid atrial firing that triggers AF is the PVs. Catheter ablation of AF depends in large part on the electrical isolation of the PVs from the remainder of the atrium. Electrophysiologic evaluation of the PVs has identified myocardial tissue that can lead to repetitive firing or even the presence of episodic reentrant activation in the veins [6].
Additionally, stretch can increase the propensity for rapid firing from the PVs as a result of stretch sensitive ion channels. [12]. It has been speculated that the mechanism of atrial stretch may help explain the association between AF and mitral regurgitation as well as various types of heart failure.
Role of atrial premature beats and other arrhythmia triggers — AF is initiated (triggered) predominantly by rapid firing from PVs. Much less commonly, AF can be triggered by non-PV sites of rapid firing (such as tissue near the PV including the Vein of Marshall, the superior vena cava, or coronary sinus) or by other types of supraventricular arrhythmias including atrioventricular nodal reentrant tachycardia (AVNRT), orthodromic AV reciprocating tachycardia, and atrial flutter [6,13-23]. In some patients, successful elimination of AF with catheter ablation requires both isolation of the PVs, as well as elimination of these non-PV triggers. (See "Catheter ablation to prevent recurrent atrial fibrillation: Clinical considerations" and "Surgical ablation to prevent recurrent atrial fibrillation".)
Role of atrial flutter and SVT — Atrial tachycardia, atrial flutter, and other supraventricular tachycardias can initiate AF in predisposed patients. The interaction between these arrhythmias and AF is not well understood, but atrial flutter and AF commonly coexist.
In some instances, elimination of atrial flutter will diminish and/or eliminate episodes of AF. Nevertheless, elimination of the right atrial reentry circuit responsible for typical flutter frequently does not eliminate the predisposition to AF that is predominately a left-atrial problem in a large number of patients. Many studies have demonstrated that patients who undergo catheter ablation of typical atrial flutter have a very high probability of developing AF over the ensuing five years. This is true regardless of whether AF had been observed prior to development of typical atrial flutter. This has clinical implications when it comes to ablation, but also has implications for anticoagulation strategies and patient follow-up. Nevertheless, for most patients, it makes sense to try to eliminate the organized supraventricular tachycardia, especially if right-sided by ablation before considering PV isolation and/or other more extensive ablation procedures to eliminate AF, as the AF may be reduced or eliminated by eliminating the other tachycardia first.
Maintenance of AF — In patients with persistent AF, the prevailing understanding of the mechanism is that, once triggered, the arrhythmia is maintained (sustained) by one or more abnormalities in the atrial tissue. This process may explain why the failure rate of PV isolation is as high as 40 to 60 percent at one year: The trigger(s) may have been treated but not the abnormalities that sustain AF once triggered (initiated).
The role of localized sources (electrical rotors and focal impulses) in the initiation and maintenance of AF was explored in the CONFIRM trial of 92 patients undergoing ablation procedures for paroxysmal or persistent (72 percent) AF [24]. Consecutive patients were prospectively treated (not randomly assigned) in a 1:2 case-cohort design with either conventional ablation at sources identified within the atria followed by conventional ablation or conventional ablation alone. Localized sources were identified in 97 percent of cases (70 percent rotors and 30 percent focal impulses) with sustained AF, each with an average of 2.1 sources. During a median of 273 days, patients treated with treatment of both sources and conventional ablation had a significantly higher freedom from AF (82.4 versus 44.9 percent).
Similar information was reported, indicating that “driver domains,” located in specific areas of the atria, act as unstable re-entry circuits that perpetuate atrial fibrillation in patients who have persistent AF [25,26].
Murine cell cultures show a differential ion channel gene expression associated with atrial tissue remodeling (ie, decreased SCN5A, CACN1C, KCND3, and GJA1; and increased KCNJ2) [27]. Fibrillatory complexity, increased in late compared with early stage cultures, was associated with a decrease in rotor tip meandering and increase in wavefront curvature.
Rotors are not the only explanation. In a study using high-density, simultaneous, biatrial, epicardial mapping of persistent and longstanding persistent AF in patients undergoing open heart surgery, several non-reentrant drivers were present in both atria in 11 or 12 patients with two to four foci per patient; foci were seen in both atria but generally in the lateral left atrial free wall, and likely acted as drivers. Reentry was not found to be the mechanism [28].
Likely, the substrate to maintain AF is a combination of reentrant activity and focal triggers. In a study of biatrial epicardial mapping of AF in sheep, wave propagation patterns were passing wave (69 percent occurrence, 68.6 percent of total time), point source (20.4, 13.1 percent), wave collision (4, 2.8 percent), reentrant wave (0.7, 6.3 percent), half-rotation (2.9, 4.4 percent), wave splitting (2.7, 4.3 percent), conduction block (0.05, 0.03 percent) and figure of eight reentry (0.05, 0.05 percent) [29]. Periods of repetitive activity were detected in the left and right atria.
The following sections describe factors that might contribute to the maintenance of AF.
Atrial remodeling — Atrial remodeling involves the concept that there are structural changes, such as fibrosis, or electrical changes, such as refractory-period dispersion or conduction display, in the atria that can predispose to the development and maintenance of AF. In some instances, structural and electrical changes occur simultaneously. These processes can facilitate or create electrical reentrant circuits or triggers that can lead to AF [13,30]. It is also well established that the presence of AF results in remodeling of the atrium over time [7]. This explains the well-established concept that AF begets AF (figure 2). Thus, the longer a patient has been in continuous AF, the less likely it is to terminate spontaneously, and harder it is to restore and maintain sinus rhythm [31].
Electrical remodeling — Paroxysmal AF commonly precedes chronic AF. It has been suggested even after only a few minutes, AF induces transient changes in atrial electrophysiology that promote its perpetuation [14]. This might occur through a tachycardiomyopathy or through "electrical remodeling" of the atria by AF, leading to a progressive decrease in atrial refractoriness [14,15]. Electrical remodeling results from the high rate of electrical activation, which stimulates the AF-induced changes in refractoriness [32]. Tachycardia-induced changes in refractoriness are spatially nonuniform and there is increased variability both within and among various atrial regions [33]. It is possible that the change in atrial refractory period observed after an episode of AF predisposes to the spontaneous recurrence of AF in the days following cardioversion.
In addition to the shortening of the refractory period, chronic, rapid, atrial pacing-induced AF results in other changes within the atria, including an increase in the expression and distribution of connexin 43 and heterogeneity in the distribution of connexin 40, both of which are intercellular gap junction proteins ("gap junctional remodeling") [16,17]; cellular remodeling is due to apoptotic death of myocytes with myolysis, which may not be entirely reversible [18]; the induction of sinus node dysfunction, demonstrated by prolonged corrected sinus node recovery time, reduced maximal heart rate in response to isoproterenol, and lower intrinsic heart rate after administration of atropine and propranolol [19]; and an increase in P wave duration and intraatrial conduction time.
A clinical study evaluated the hypothesis of electrical remodeling by the use of atrial pacing-induced AF in patients with a history of supraventricular tachycardia [20]. AF significantly shortened the right-atrial effective refractory period after only a few minutes, and temporal recovery of the refractory period occurred over about eight minutes. Upon termination of AF, there was an increased propensity for the induction of another episode of AF that decreased with increasing time after the initial AF reversion. The second also tends to last longer than the first.
The time to recurrence was also evaluated in a review of 61 patients who had daily electrocardiogram (ECG) recordings using transtelephonic monitoring: 57 percent had recurrent AF during the first month after cardioversion, with a peak incidence during the first five days [21]. Among patients with recurrence, there was a positive correlation between the duration of the shortest coupling interval of atrial premature beats after cardioversion, which correlates with the refractory period and the timing of recurrence (figure 3). (See "Atrial fibrillation: Cardioversion to sinus rhythm".)
In contrast to the normal situation in which the atrial refractory period shortens with an increase in rate (as in AF) and prolongs when the rate decreases, the refractory period fails to lengthen appropriately at slow rates (eg, with return to sinus rhythm) in patients with acute or chronic AF. The duration of AF has no significant impact upon the extent of these electrophysiologic changes [22].
Atrial electrical remodeling is reversed gradually after the restoration of sinus rhythm [23,34]. This may be one of the explanations for the early or immediate return of AF after cardioversion. In one study of 25 patients, the atrial refractory period increased and the adaptation of atrial refractoriness to rate was normal by four weeks after cardioversion [23]. In another report of 38 patients, the atrial refractory period increased by one week, with some variation in different regions of the atrium [34]. This observation has important clinical implications.
The mechanism for electrical remodeling and shortening of the atrial refractory period is not entirely clear; a possible explanation is ion-channel remodeling, with a decrease in the protein content of the L-type calcium channel [35]. Support for this comes from an animal study in which verapamil, an L-type calcium antagonist, prevented electric remodeling of short-duration AF (one day or less) and hastened complete recovery, without affecting inducibility of AF [36]. Similar findings have been noted in humans as verapamil, but not procainamide, prevented remodeling when given prior to the electrophysiologic induction of AF [37]. Oral diltiazem is also effective in some patients [38], while beta blockers had no effect on electrical remodeling in an animal model [32].
In comparison, cytosolic calcium overload, induced by hypercalcemia or digoxin, which increases the intracellular concentration of calcium by activating the sodium-calcium exchanger, enhances electrical remodeling [36,39,40]. The effect of digoxin, which is not due to its vagotonic activity, is associated with an increase in the inducibility and duration of AF [40].
Calcium leak from the sarcoplasmic reticulum may trigger and maintain AF. It is known that protein kinase A (PKA) hyperphosphorylation of the cardiac ryanodine receptor (RyR2), resulting in dissociation of the channel-stabilizing subunit calstabin2, causes sarcoplasmic reticulum (SR) calcium leak in failing hearts. This phenomenon seems to be involved in triggering ventricular arrhythmias.
Using similar logic, these proteins were investigated in atrial tissues from both dogs and humans with AF [41]. Atrial tissue in those with AF showed a significant increase in PKA phosphorylation of RyR2 and a decrease in calstabin2 binding to the channel. Channels isolated from dogs with AF had an increased open probability under conditions simulating diastole compared with channels from control hearts, suggesting that these AF channels could predispose to a diastolic SR calcium leak. The conclusion was that SR calcium leak due to RyR2 PKA hyperphosphorylation may play a role in the initiation and/or maintenance of AF. Other studies also suggest that RyR2 receptor-mediated calcium leak drives progressive development of an atrial fibrillation substrate in a transgenic mouse model [42,43].
The effects of calcium overload are quite complex. It is likely that triggers and substrates initiate short episodes of AF that then lead to calcium overload and over a period of minutes there is activation of the I Ca,L current that increases I K1, decreases I Na, increases IKACh, and decreases ITO. This can affect the action-potential duration and allow for more reentry to occur. As reentry occurs, the substrate changes and there is remodeling through calcium handling abnormalities as well as mRNA transcription [31], and ultimately perhaps with protein decrease, changes in connexons, including, Cx40, that can affect conduction. The calcium-handling abnormalities can also lead to hypocontractility and atrial dilatation, thereby affecting even more the possibility of developing AF [31].
Both animal and human studies suggest that angiotensin II is involved in electrical and atrial myocardial remodeling [44,45] (see "Actions of angiotensin II on the heart"). In an animal model, inhibition of angiotensin II with captopril or candesartan prevented shortening of the atrial effective refractory period and atrial electrical remodeling during rapid atrial pacing [44], while atrial tissue obtained during open heart surgery from patients with AF revealed downregulation of AT1 receptor proteins and upregulation of AT2 receptor [45]. The potential clinical importance of these changes is illustrated by the observations that angiotensin converting enzyme (ACE) inhibitors reduce the incidence of AF in patients with left ventricular dysfunction after myocardial infarction [46] and in patients with chronic left ventricular dysfunction due to ischemic heart disease [47]. (See "Angiotensin converting enzyme inhibitors and receptor blockers in acute myocardial infarction: Clinical trials".)
Another possible contributor to electrical remodeling and shortening of the atrial refractory period is atrial ischemia, which activates the sodium/hydrogen exchanger. The intravenous administration of HOE 642, a selective inhibitor of this sodium proton pump, to dogs undergoing rapid atrial pacing resulted in the lengthening of atrial refractoriness after one hour, while control dogs showed effective refractory-period shortening greater than 10 percent [48].
Role of the autonomic nervous system — It is increasingly well recognized that the autonomic nervous system plays an important role in the development and maintenance of AF [49-51]. Both the parasympathetic and the sympathetic nervous system have been implicated in the genesis of AF [52-54]. Early epidemiologic studies suggested that exercise-induced AF may be sympathetically driven [55]. The parasympathetic nervous system is thought to contribute to AF in young patients with no structural heart disease [56].
The following studies further support a role for the autonomic nervous system in the genesis and/or maintenance of AF:
●Early studies suggested that exercise-induced AF may be sympathetically driven [53,55].
●The parasympathetic nervous system may contribute to AF in young patients with no structural heart disease [56].
●Animal studies show that vagal stimulation contributes to the genesis of AF by non-uniform shortening of atrial effective refractory periods, thereby setting up substrate for reentry. Vagal stimulation can also lead to the emergence of focal triggers in the atrium [57-59].
●PV ectopic foci also appear to be at least partially modulated by autonomic signaling, with sympathetic stimulation with isoproterenol being frequently utilized to “bring out” these triggers in patients undergoing ablation for AF [60].
●Clinical studies using heart rate variability analysis in patients with focal AF suggest that fluctuation in autonomic tone may be a major determinant of AF in patients with focal ectopy originating from the PVs [61]. Related studies have also demonstrated a change in heart rate variability after PV ablation [62], further suggesting that PV triggers may be at least partially modulated by autonomic activity. Another study showed that the occurrence of paroxysmal AF greatly depends on variations of the autonomic tone, with a primary increase in adrenergic tone followed by an abrupt shift toward vagal predominance [63].
●It is also well known that Bezold-Jarisch-like “vagal” reflexes can be elicited during radiofrequency ablation and occur in and around the PVs. It has been suggested that elimination of these vagal reflexes during ablation may improve efficacy of AF ablation procedures [64].
●Vagal responsiveness also appears to decrease following ablation in the left atrium [65]. In some series, adding ganglionated plexi (GP) ablation to PV isolation appears to increase ablation success for AF [12].
●Data suggest that areas in the atrium demonstrating complex fractionated atrial electrograms (CFAE) may represent a suitable target site for ablation; although several studies have reported that ablation at these sites may increase the efficacy of PV isolation procedures [66,67], enthusiasm for this approach has fallen over time. One possible explanation for the improvement in ablation success reported in these trials is that several CFAE sites anatomically overlie fat pads containing GPs [18,68]. As indicated above, autonomic denervation performed by GP ablation is thought to improve efficacy of AF ablation. (See "Catheter ablation to prevent recurrent atrial fibrillation: Clinical considerations" and "Catheter ablation to prevent recurrent atrial fibrillation: Technical considerations", section on 'Complex fractionated atrial electrograms'.)
●Anatomic studies of the autonomic innervation of the atria also indicate that the PVs and posterior left atrium (PLA) have a unique autonomic profile with a rich innervation from sympathetic and parasympathetic nerves [53,69-73]. The autonomic nervous system may also be playing a role in the genesis of AF in diseased hearts [53,74,75].
●In a study of 40 patients with paroxysmal AF scheduled to undergo catheter ablation, individuals were randomly assigned to noninvasive transcutaneous low-level stimulation of the tragus (the anterior protuberance of the ear where the auricular branch of the vagus nerve is accessible) or to sham stimulation for one hour. Compared to control, low-level stimulation suppressed AF as measured by the decreased duration of atrial pacing-induced AF and an increased AF cycle length [76].
Studies suggest that the parasympathetic and sympathetic nervous system may also be playing a role in creation of AF substrate in the setting of congestive heart failure [74,75].
Role of fibrosis — The development of AF invokes atrial remodeling processes that involve electrophysiological and structural alterations that serve to maintain, promote, and propagate AF. In addition to electrophysiological alterations, such as shortening of the atrial action potential, increased dispersion of refractoriness, and conduction velocity shortening, morphological changes consist of fibrosis, hypertrophy, necrotic and apoptotic cell loss, and dilation [77].
Of these, fibrosis is considered especially important in the creation of AF substrate, especially in the setting of chronic atrial dilatation caused by heart failure. A canine model of heart failure has demonstrated a progressive increase in AF inducibility with increasing fibrosis [78]. An increase in conduction heterogeneity noted in this model is thought to play a major role in the creation of reentrant circuits in the dilated atria. Patients with AF also display increased atrial fibrous tissue content, along with increased expression of collagen I and III [79], as well as up-regulation of MMP-2 protein, and down-regulation of the tissue inhibitor of metalloproteinase, TIMP-1 [79]. Expression of the active form of MMP-9 and of monocyte chemoattractant protein-1, an inflammatory mediator, is increased in AF patients [80]. The left atrial free wall around the PV area presents particularly strong interstitial fibrotic changes [81-83].
Although the underlying molecular mechanisms that lead to the development of atrial fibrosis are complex, work suggests that the TGF-β pathway may be an important contributor to the development of fibrosis (especially in the setting of increasing atrial stretch/dilatation resulting from congestive heart failure) [84-86].
Role of inflammation and oxidative stress — Emerging evidence suggests a significant role of inflammation in the pathogenesis of AF [87]. Evidence includes elevated serum levels of inflammatory biomarkers in patients with AF, the expression of inflammatory markers in atrial tissue from AF patients, and beneficial effects of antiinflammatory drugs in the setting of experimental AF [88]. Inflammation is suggested to be linked to various pathological processes, such as oxidative stress, apoptosis, and fibrosis that promote the creation and perpetuation of AF substrate. Several of the downstream effects of inflammation in the heart are thought to be mediated by oxidative stress [89].
Indeed, studies in patients with AF demonstrate increased generation of reactive oxygen species (ROS) in the fibrillating atrium compared with normal atria [90,91]. Several major enzymatic sources of ROS have been implicated in AF. Of these, NAPDH oxidase (specifically its NOX2 isoform) has been shown to be elevated in humans with AF in a variety of studies [92,93]. Other sources of ROS implicated in AF include uncoupled nitric oxide synthase [94] and xanthine oxidase [95]. In addition to the increase in ROS noted in tissue from patients with AF, experimental evidence suggests that ROS may be implicated not only in promoting AF but also in maintaining atrial arrhythmia. The administration of antioxidants such as vitamin C or statins (which are known to have pleiotropic antioxidant effects) decreased AF inducibility in canine models of tachypacing-induced AF [96,97]. Antioxidants such as vitamin C and n-acetylcysteine have been administered to patients undergoing cardiac surgery and have been shown to decrease postoperative AF [98,99]. These early results are encouraging and warrant further investigation of inflammation and oxidative stress as viable therapeutic targets in patients with AF.
Reentrant mechanism — Maps of AF in animals and humans suggest that this arrhythmia is caused by multiple wandering wavelets (figure 4), and these may be due to heterogeneity of atrial refractoriness and conduction. In addition, the response of atrial activity to adenosine infusion suggests a reentrant rather than a focal mechanism [100]. Adenosine increases the inward potassium rectifier current, which shortens refractory periods and would accelerate reentrant circuits. In contrast, this effect would slow an automatic or triggered focus. In a series of 33 patients with AF undergoing electrophysiology study, adenosine increased the dominant frequencies, supporting reentrant rather than focal sources for the perpetuation of AF.
It has been suggested that at least four to six independent wavelets are required to maintain AF [101]. These wavelets rarely reenter themselves but can re-excite portions of the myocardium recently activated by another wavefront, a process called random reentry [7,102-104]. As a result, there are multiple wavefronts of activation that may collide with each other, extinguishing themselves or creating new wavelets and wavefronts, thereby perpetuating the arrhythmia (figure 5).
The reentrant circuits are therefore unstable; some disappear, while others reform. These circuits have variable but short cycle lengths, resulting in multiple circuits to which atrial tissue cannot respond in a 1:1 fashion. As a result, functional block, slow conduction, and multiple wave fronts develop [104].
Patients with AF may have increased dispersion of refractoriness. This correlates with enhanced inducibility of AF and spontaneous episodes [105] likely related to unstable reentry circuits. Some patients have site-specific dispersion of atrial refractoriness and intraatrial conduction delays resulting from nonuniform atrial anisotropy [106]. This appears to be a common property of normal atrial tissue, but there are further conduction delays to and within area surrounding the AV node in patients with induced AF, suggesting an important role for the low right atrium in the genesis of AF.
Abnormalities in restitution as well as the spatial distribution of such abnormalities can be related to the persistence of AF. In one study, monophasic action potential recordings were evaluated in patients with AF [107]. The action potential duration was plotted as a function of the preceding diastolic interval, and the slope of the action potential duration versus the diastolic interval (the restitution curve) was determined. If the slope was greater than one, oscillations occurred that may cause localized conduction delay or block resulting in a wave break giving rise to atrial fibrillation.
These different patterns of conduction are reflected in the morphology of electrograms recorded with mapping during induced AF. Single potentials were indicative of rapid uniform conduction, short double potentials indicated collision, long double potentials were indicative of conduction block, while fragmented potentials were markers for pivoting points or slow conduction (figure 6) [108,109].
Sites of fragmented potentials or complex fractionated atrial electrograms are potential targets for radiofrequency ablation to terminate AF as they may represent critical areas from which AF originates and perpetuates. (See "Catheter ablation to prevent recurrent atrial fibrillation: Clinical considerations".)
This phenomenon has been termed “microreentry” to distinguish it from classic reentry in which the same reentrant pathway is repetitively traversed. The impulse may circulate around a central line of functional block, so-called leading circle reentry; this type of reentry tends not to be stable but rather to drift through the atria until it is extinguished. The perpetuation of AF may also depend importantly upon macroreentry around natural orifices and structures in the atrium, which provides a rationale and anatomic landmarks for ablative treatment. The collision of wavefronts cancels many atrial depolarizations that might otherwise reach the AV node, resulting in a slower heart rate than might otherwise have occurred (figure 7A-B).
Although multiple wandering wavelets probably account for the majority of AF, one study reported nine patients in whom a single, rapidly firing focus was identified with electrophysiologic mapping [110]. Organized and rapid atrial activity with a centrifugal and consistent pattern of atrial activation resulted from this focus, but it fired irregularly with striking and abrupt changes in atrial cycle lengths. In most of the patients, the focus was near the ostia of great vessels and was amenable to radiofrequency ablation (figure 8 and figure 9).
Small reentrant sources, called rotors, may drive or maintain AF in some cases. These rotors result in a hierarchical distribution of frequencies throughout the atria that may be identified with spectral analysis of intracardiac recordings. Ablation of such sites has terminated paroxysmal AF, suggesting that they may play an important role [111], but it is not clear that the rotors are responsible for AF or are fixed in most instances. AF may be chaotic and have wavelets and rotors that are secondary rather than the predominant cause of AF [112]. However, antral pulmonary venous reentrant and focal drivers may be responsible for AF [26]. The complexity of such drivers increase with prolonged AF. These sites are often localized near the PV orifices in patients with paroxysmal AF, and are more often localized to the left or right atria in patients with chronic AF [100].
The fibrillating atrium cannot be captured by pacing when the atrial electrograms are disorganized. This observation supports the presence of microreentry, since there is no excitable gap (or it is very small) to permit capture. However, when type I (figure 9) AF (which has organized atrial electrograms) is induced by rapid atrial pacing, the fibrillating atrium can be captured with rapid atrial pacing, suggesting the presence of an excitable gap [113].
ROLE OF THE ATRIOVENTRICULAR NODEThe atrioventricular (AV) node regulates the number of atrial impulses that reach the ventricle. The ventricular rate in atrial fibrillation (AF) is typically irregularly irregular, with a ventricular rate that may be slow, moderate, or rapid depending on the capacity of the AV node to conduct impulses. The rate of AV nodal conduction is dependent upon multiple factors, including electrical properties of the node and the influence of the autonomic nervous system [114]. In addition, the use drugs such as digoxin, calcium channel blockers, or beta blockers may influence AV nodal function. There also may be a circadian rhythm for both AV nodal refractoriness and concealed conduction, accounting for the circadian variation in ventricular response rate [115].
AV nodal tissue consists of so-called "slow response" fibers, which depend on a mixed calcium/sodium current. This current is often called the inward calcium current, since in a normal physiologic environment, the ions are almost exclusively calcium. The mixed current uses a kinetically slow channel and is responsible for phase 0 depolarization. (See "Cardiac excitability, mechanisms of arrhythmia, and action of antiarrhythmic drugs".)
These characteristics lead to properties that are quite different from "fast-response" tissue in the atria, which as noted above, depend on an inward sodium current that uses a kinetically fast channel for phase 0 depolarization [8,116]:
●Partial and complete reactivation returns only 100 msec or more after return to the diastolic potential (versus 10 to 50 msec in the atria).
●The refractory period changes little as a function of rate.
●Conduction velocity is relatively slow, ranging from 0.01 to 0.1 m/sec.
●Unlike tissue generating a fast action potential that has an all-or-none response (ie, the velocity of impulse conduction is similar at all stimulation rates until block occurs), tissue that generates a slow action potential exhibits a graded or decremental response, in which the velocity of impulse conduction slows as the stimulation rate increases.
●As noted above, the ventricular rate usually ranges 90 and 170 beats/min. Ventricular rates below 60 beats/min are seen with AV nodal disease, drugs that affect conduction, and high vagal tone as can occur in a well-conditioned athlete. Ventricular rates above 200 beats/min suggest catecholamine excess, parasympathetic withdrawal, or the existence of an accessory bypass tract as occurs in the preexcitation syndrome. The QRS complexes are widened in the last setting and must be distinguished from a rate-related or underlying bundle branch block.
In the classical view, the AV node is bombarded by impulses from the fibrillating atria. Some impulses traverse the AV node and reach the specialized infranodal conduction system and then the ventricles. However, most atrial impulses penetrate the AV node from varying distances and then are extinguished when they encounter the refractoriness of an earlier wavefront; this phenomenon of concealed conduction in turn creates a refractory wave that affects succeeding impulses. The failure of the refractory period to shorten with increasing rate (as occurs in the atria) further decreases the likelihood of an impulse traversing the AV node.
Anatomically distinct AV nodal inputs, called the slow and fast pathways, are involved in the ventricular response to AF. The importance of these pathways has been demonstrated in radiofrequency ablation studies in which ablation reduced the number of beats that successfully reached the infranodal conduction system and the ventricles [117-120]. (See "Atrioventricular nodal reentrant tachycardia".)
In addition to its intrinsic properties, the AV node is richly supplied and affected by both components of the autonomic nervous system. AV conduction is enhanced and refractoriness reduced by the sympathetic fibers, and conduction reduced and refractoriness prolonged by the parasympathetic fibers.
The net effect of the electrophysiologic properties of the AV node is that the rate of conduction into the specialized infranodal conduction system is (fortunately) much slower than the rate of the fibrillating atria. In some cases, the high degree of refractoriness in the AV node with AF results in high-grade or third-degree block. In this setting, the pacemaker that controls the ventricles is below the AV node. (See "The electrocardiogram in atrial fibrillation", section on 'Effect of high degrees of atrioventricular nodal block and exit block on ventricular response'.)
In patients with the preexcitation syndrome, the AV node is bypassed by "fast-response" tracts, which activate and reactivate much faster than the AV node and are therefore capable of rapid conduction. The development of AF in such a patient can result in very rapid transmission of atrial impulses to the ventricles [120] and can rarely cause ventricular fibrillation [15]. (See "The electrocardiogram in atrial fibrillation", section on 'Atrial fibrillation with an atrioventricular bypass tract'.) It is also important to recognize that the presence of an accessory pathway can increase the propensity for development of AF. In patients with AF who have Wolff-Parkinson-White (WPW) syndrome, catheter ablation of the accessory pathway is indicated to lower the sudden death risk but also to decrease the probability of recurrent AF.
Unexpected ventricular rates — The ventricular response to AF characteristically is irregularly irregular although it may appear regular in the presence of complete AV block. The usual ventricular rate in AF is between 90 and 170 beats per minute in the absence of AV node disease, drugs that affect conduction, or enhanced vagal inputs. Ventricular rates that are clearly outside this range suggest some concurrent problem:
●A ventricular rate below 60 beats per minute, in the absence of AV nodal blocking agents, suggests AV nodal disease that may be associated with the sinus node dysfunction. (See "Sinus node dysfunction: Epidemiology, etiology, and natural history".)
●A ventricular rate above 170 beats per minute suggests thyrotoxicosis, catecholamine excess, parasympathetic withdrawal, or the existence of an accessory bypass tract in the preexcitation syndrome. (See "Epidemiology of and risk factors for atrial fibrillation".)
SPECIFIC CLINICAL SITUATIONS
Late recurrent AF after catheter ablation — The etiology of late recurrent atrial fibrillation (AF) following pulmonary vein isolation (PVI) has been debated. In some cases, triggering foci outside of the PVs may initiate AF [121-124]. Alternatively, persistence of the substrate for maintaining AF (abnormal electrical properties of the atria themselves) may be more important than the triggering foci, especially in chronic AF.
However, there is increasing evidence that when AF does recur late after PVI, it often represents incomplete electrical isolation of the PVs, either due to resumption of conduction across the ablation scar or to residual conduction in PVs that were not successfully ablated. Most [125-128], but not all [129], studies of the former mechanism support the hypothesis that resumption of PV-left atrial (LA) conduction is associated with an increased risk of recurrent AF. However, recurrent conduction across ablated lesions is more common than clinically evident recurrent AF [127,130].
Pre-existing LA scarring may predispose patients to late recurrence. In a series of 700 consecutive patients undergoing first-time PVI, scarring was detected in 6 percent [131]. These patients had a much higher rate of recurrence than those without scarring (57 versus 19 percent).
Possible causes of scarring include atrial remodeling and inflammation. The patients with scarring had significant elevations in serum C-reactive protein (CRP) compared to those without scarring (5.9 versus 0.31 mg/L). This is consistent with other studies showing a relationship between serum CRP and AF [132]. (See "Epidemiology of and risk factors for atrial fibrillation", section on 'Inflammation and infection'.)
After cardiac surgery — AF occurs frequently (approximately one of four patients) after cardiac surgery. Nonuniform atrial conduction is greatest on days two and three in this setting, and the longest atrial conduction time is greatest on day three after open heart surgery; these abnormalities coincide with the time of greatest risk for AF [133]. The degree of atrial inflammation after surgery in dogs was associated with a proportional increase in the inhomogeneity of atrial conduction and in the duration of AF; antiinflammatory therapy decreased the inhomogeneity [134]. Nevertheless, the mechanism of AF in the postoperative period is likely multifactorial. It is important to note that in most of the patients, especially those without a prior history of AF, that the AF is self-limited, and antiarrhythmic drug therapy can usually be stopped two to three months following surgery when the inflammation has subsided. (See "Atrial fibrillation and flutter after cardiac surgery".)
Hyperthyroidism — It is well established that hyperthyroidism can increase susceptibility to development of AF. As a consequence, all patients with new onset AF should have some measure of thyroid function tested. Successful treatment of the hyperthyroidism often results in elimination of the AF.
Obesity — Obesity has been associated with AF and it is possible that both are related mechanistically [135]. In a sheep model, weight gain was associated with increased left atrial volume, fibrosis, inflammatory infiltrates, and lipidosis. There was reduced conduction velocity in atrial tissue and increased inducible and spontaneous AF with obesity. Atrial endothelin-A and -B receptors, endothelin-1, atrial interstitial and cytoplasmic transforming growth factor beta1, and platelet-derived growth factor were higher with obesity. In a clinical study of 110 patients undergoing AF ablation versus 20 reference patients without AF, pericardial fat volumes were associated with AF, its chronicity, and its symptom burden. Pericardial fat predicted AF recurrence post-ablation [136]. Associations persisted after adjusting for body weight but body mass index was not associated with these outcomes in multivariate-adjusted models. In another report [137], weight management with subsequent weight loss was associated with improved AF symptom burden scores, symptom severity scores, number of episodes, and cumulative duration of AF. This preliminary information does not yet prove that obesity causes AF by any specific mechanism. In a study of atrial sheep myocytes, acute, short-term incubation in free fatty acids resulted in no differences in passive or active properties of isolated left atrial myocytes but stearic acid reduced membrane capacitance and abbreviated the action potential duration, likely due to a reduction of the L-type calcium and of the transient outward potassium currents [138].
GENETICS OF AFOver the last decade, a preponderance of evidence suggests a large genetic contribution to atrial fibrillation (AF) [139,140]. Having a family member with AF is associated with a 40 percent increased risk for the arrhythmia [141]. Initially, traditional genetic techniques such as linkage analysis led to the discovery of rare, monogenic causes of AF. The first such study identified a genetic locus for AF using a series of related families with early onset AF [142]. A later study identified the first gene for familial AF [143]. Using a large Chinese kindred with autosomal dominant AF, they found a gain-of-function mutation in KCNQ1 (the gene encoding the α subunit of the potassium channel current, IKs). Since then, several additional gain-of-function variants have been identified in KCNQ1 [144,145]. In addition to KCNQ1, mutations have been identified in other potassium channels genes, including KCNA5 [146], KCND3 [147], and KCNJ2 [148], and accessory subunits KCNE1 [149], KCNE2 [150], KCNE3 [151], and KCNE5 [152,153].
The majority of these functionally validated, AF-associated potassium channel variants have a gain-of-function channel, with an expected shortening of the atrial action potential duration and atrial refractory period. Variation in sodium channel subunits has also been identified as an important factor in the development of familial AF, with AF-causing variants observed in both the major cardiac sodium channel alpha subunit SCN5A [154] and its associated beta subunits [155,156]. Several variants have also been identified in genes that do not directly alter the atrial action potential, but instead would be expected to cause AF through alternative mechanisms, eg, somatic mutations in GJA5, which encodes the gap junctional protein; connexin 40, a frameshift mutation that resulted in early truncation of NPPA [157], which encodes for the precursor for atrial natriuretic peptide; and genetic variation in several developmentally related cardiac transcription factors, ie, NKX2.5, PITX2, GATA4, GATA5, and GATA6[156,158,159].
Genome-wide association studies (GWAS) have been used to identify genetic loci associated with AF. GWAS rely on the unbiased comparison of common single-nucleotide polymorphisms (SNPs) throughout the genome, with SNPs occurring with different frequency in individuals with a disease versus controls being used to localize disease-related genetic loci. The first GWAS performed for AF identified a region on chromosome 4q25, which was associated with AF in those of European and Asian descent [160]. Subsequently, these findings were broadly replicated in individuals of European, Asian, and African descent [161,162]. Genetic variants on chromosome 4q25 that are most significantly associated with AF reside about 150 kilobases upstream of the nearest gene PITX2. PITX2 encodes the paired-like homeodomain transcription factor 2, which helps determine cardiac laterality, suppresses the default expression of a sinoatrial nodal gene programme in the left atrium, and encodes the pulmonary venous myocardium [163]. In addition, PITX2 is associated with formation of the pulmonary veins. These findings are particularly interesting in light of the fact that AF triggers frequently arise in the pulmonary veins. In addition to the role of PITX2 in development, studies demonstrate a role for the PitX2c transcript in expression of gene-encoding ion channels, calcium cycling proteins, and gap junctions; these direct electrophysiological influences likely lead to formation of substrate for triggered activity as well as reentry [164].
Related analysis identified the same genomic region as being associated with an increased risk of cardioembolic stroke [156,165] and a prolonged PR interval [166]. To date, GWAS have identified 14 genomic regions of susceptibility for AF, with 17 independent signals at these loci [167]. These include the ZFHX3 gene that encodes a zinc finger homeobox transcription factor [168], the KCNN3 gene that encodes the SK3 potassium channel [169], and the PRRX1 gene that encodes a member of the paired-related homeobox gene family [168]. Whole exome and genome sequencing has been increasingly used to identify rare variants associated with AF [156]. For example, Oleson et al reported a much higher prevalence of rare variants in genes associated with AF (KCNQ1, KCNH2, SCN5A, KCNA5, KCND3, KCNE1, 2, 5, KCNJ2, SCN1-3B, NPPA, and GJA5) in early onset, lone AF patients than in the background population [170]. This approach is beginning to identify rare candidate variants in genes not previously linked to other types of Mendelian diseaseand thus may offer new insights into AF pathogenesis and disease pathways that could ultimately provide novel therapeutic targets for this common condition.
++++++++++++++
Changes in the anatomy and electrophysiology of the atrial myocardium causes atrial fibrilations
Atrial fibrillation (AF) is associated with elevation in atrial pressure, or infiltration or inflammation of the atria are often seen.
Hypertensive heart disease and coronary heart disease (CHD) are the most common underlying chronic disorders in patients with atrial fibrillation (AF) in developed countries. Rheumatic heart disease, although now uncommon in developed countries, is associated with a much higher incidence of AF. Paroxysmal AF (PAF) is associated with the same disorders as chronic (permanent) AF. (See "Paroxysmal atrial fibrillation".)
Hypertensive heart disease — In a longitudinal study of male air crew recruits, a history of hypertension increased the risk of developing AF 1.42-fold [22]. Although this is a relatively small increase in risk, the high frequency of hypertension in the general population results in hypertensive heart disease being the most common underlying disorder in patients with AF [20].
Coronary disease — AF is not commonly associated with CHD unless it is complicated by acute myocardial infarction (MI) or heart failure (HF). AF occurs transiently in 6 to 10 percent of patients with an acute MI, presumably due to atrial ischemia or atrial stretching secondary to HF [24-27]. These patients have a worse prognosis that is mostly due to comorbidities such as older age and HF. (See "Supraventricular arrhythmias after myocardial infarction", section on 'Atrial fibrillation'.)
The incidence of AF is much lower in patients with chronic stable CHD [28,29]. In the Coronary Artery Surgical Study (CASS), which included over 18,000 patients with angiographically documented coronary artery disease, AF was present in only 0.6 percent [28]. These patients probably had chronic AF; the prevalence of PAF may be higher. AF was associated with age greater than 60, male sex, mitral regurgitation (MR), and HF; there was no association between AF and the number of coronary arteries involved.
Valvular heart disease — Almost any valvular lesion that leads to significant stenosis or regurgitation is associated with the development of AF. The following are representative frequencies:
●In a review of 89 patients with mitral valve prolapse (and grade 3 or 4 MR) and 360 with flail leaflets, the rate of development of AF was about 5 percent per year with both types of lesions [30]. The major independent risk factors were age ≥65 years and baseline left atrial dimension ≥50 mm.
●Rheumatic heart disease is now uncommon in developed countries. It is, however, associated with a high prevalence of AF [31,32]. In a study of approximately 1100 patients with rheumatic heart disease, the prevalence varied with the type of valve disease [32]:
•Mitral stenosis (MS), MR, and tricuspid regurgitation – 70 percent
•MS and MR – 52 percent
•Isolated MS – 29 percent
•Isolated MR – 16 percent
Heart failure — AF and HF often occur together, and each may predispose to the other [33]. Among patients with HF, the prevalence of AF is variable, depending in part upon the severity of HF. Issues related to AF in patients with HF and cardiomyopathy are discussed in detail separately. (See "The management of atrial fibrillation in patients with heart failure".)
Hypertrophic cardiomyopathy — AF has been reported in 10 to 28 percent of patients with hypertrophic cardiomyopathy [34-36]. The prognostic importance of AF in these patients is unclear, with some reports showing a worse prognosis [36] and others no increase in mortality [35]. (See "Hypertrophic cardiomyopathy: Prevalence, pathophysiology, and management of concurrent atrial arrhythmias".)
Congenital heart disease — AF has been reported in approximately 20 percent of adults with an atrial septal defect [37]. However, the incidence of AF is related to age, ranging in one series from 15 percent for those aged 40 to 60, to 61 percent for those over the age of 60 [38].
AF and/or atrial flutter also occurs in other forms of congenital heart disease that affect the atria, including Ebstein anomaly and patent ductus arteriosus, and after surgical correction of some other abnormalities, including ventricular septal defect, tetralogy of Fallot, pulmonic stenosis, and transposition of the great vessels.
Venous thromboembolic disease — Venous thromboembolic disease, which includes deep vein thrombosis and pulmonary embolism, is associated with an increased risk of AF. The mechanism is not known but has been speculated to be related to the increase in pulmonary vascular resistance and cardiac afterload, which may lead to right atrial strain [39,40]. (See "Clinical manifestations and diagnosis of chronic thromboembolic pulmonary hypertension", section on 'Diagnostic evaluation'.)
The incidence of AF in patients with acute or chronic venous thromboembolic disease has not been well studied. It has been reported to be in the 10 to 14 percent range in patients with documented pulmonary embolism [41,42]. The impact of incident venous thromboembolism (VTE) on the future risk of AF was evaluated in a prospective population-based study of nearly 30,000 individuals of whom 1.8 percent had an incident VTE event and 5.4 percent were diagnosed with AF during 16-year follow-up [40]. The risk of AF was higher in those with VTE than in those without after multivariable adjustment (hazard ratio 1.63, 95% CI 1.22-2.17). This risk was particularly high in the first six months after the VTE event.
Other types of cardiopulmonary disease — AF is associated with a variety of other types of cardiopulmonary disease:
●AF also occurs in chronic obstructive pulmonary disease [43,44], peripartum cardiomyopathy [45], lupus myocarditis [46], and both idiopathic and uremic pericarditis [47,48].
●There is a possible causal relationship between obstructive sleep apnea (OSA) and AF [49-52]. In a series of 39 patients diagnosed with both PAF and OSA, patients receiving treatment with continuous positive pressure ventilation had a lower incidence of AF recurrence at 12 months (42 versus 82 percent for patients who were not treated) [49]. In another observational study, the incidence of OSA was compared between 151 patients referred for cardioversion for AF and 312 controls without AF referred for general cardiology evaluation [50]. OSA was significantly more common in the patients with AF than in the control group (49 versus 32 percent). Finally, preoperative sleep studies were performed in a series of 121 patients referred for coronary artery bypass surgery [51]. Postoperative AF was significantly more common among the 49 patients with an abnormal sleep study (39 versus 18 percent in patients with normal sleep studies). (See "Obstructive sleep apnea and cardiovascular disease in adults", section on 'Atrial fibrillation'.)
Obesity — Obese individuals (body mass index [BMI] >30 kg/m2) are significantly more likely to develop AF than those with a normal BMI (<25 kg/m2) [53,54]. In the Framingham Heart Study, every unit increase in BMI was associated with an approximate 5 percent increase in risk [55]. (See "Overweight and obesity in adults: Health consequences".)
A primary mechanism for the role of obesity may be an increase in the size of the left atrium. Increased left atrial pressure and volume, often associated with diastolic dysfunction, as well as a shortened effective refractory period in the left atrium and in the proximal and distal pulmonary veins have been identified as potential factors facilitating and perpetuating AF in obese patients [56]. Inflammation and pericardial fat may also play a role [53]. (See "Mechanisms of atrial fibrillation" and 'Other factors' below.)
There is some evidence to suggest that long-term weight loss is associated with a reduction of AF burden [54,57].
Diabetes — In a study of over 4700 individuals without valvular heart disease in the Framingham Heart Study, the presence of diabetes was associated with a significantly increased risk for the development of AF in multivariate analysis (odds ratio 1.1 for men and 1.5 for women) [58]. Increased left ventricular mass and increased arterial stiffness have been put forth as possible mechanisms [59].
Metabolic syndrome — As discussed above, the presence of hypertension, diabetes, or obesity is associated with an increased likelihood of the development of AF. The metabolic syndrome includes these three, as well as dyslipidemia. (See "The metabolic syndrome (insulin resistance syndrome or syndrome X)", section on 'Definition'.)
The potential relationship between the metabolic syndrome and the development of AF was evaluated in a prospective, observational cohort study of 28,449 Japanese citizens [60]. Using the 2005 criteria for the metabolic syndrome approved by the American Heart Association and the National Heart, Lung, and Blood Institute, 4544 individuals met criteria for the metabolic syndrome at baseline [61]. During a mean follow-up for 4.5 years, AF developed in 265 patients. The risk of developing AF was significantly greater in those individuals with the metabolic syndrome (hazard ratio 1.61, 95% CI 1.21-2.15), as well as in those with individual components of hypertension, obesity, low high density lipoprotein cholesterol and impaired glucose tolerance, but not elevated triglycerides.
Chronic kidney disease — Chronic kidney disease (CKD) increases the risk of the development of AF. The following two prospective, cohort studies are representative:
●In a study of 235,818 individuals, the hazard ratio for the development of AF was 1.32 for patients with estimated glomerular filtration rates (eGFRs) of 30 to 59 mL/min/1.73m2compared with those with normal renal function [62].
●The relationship between CKD and AF was evaluated in a report of 10,328 individuals free of AF participating in the Atherosclerosis Risk in Communities (ARIC) study who had a baseline cystatin C-based estimated glomerular filtration rate (eGFRcys) [63]. Compared with individuals with eGFRcys ≥90 mL/min/m2, the multivariable hazard ratios for the development of AF were significantly increased at 1.3, 1.6, and 3.2 in those with eGFRcys of 60 to 89, 30 to 59, and 15 to 29 mL/min/m2, respectively, during a median follow-up of 10.1 years. In addition, macroalbuminuria and microalbuminuria were significantly associated with higher AF risk.
The incidence and prevalence of AF in patients with CKD are presented separately. (See "Management of thromboembolic risk in patients with atrial fibrillation and chronic kidney disease", section on 'Prevalence and incidence'.)
POTENTIALLY REVERSIBLE TRIGGERSThe medical conditions listed above, which are associated with an increased risk of the development of atrial fibrillation (AF), are chronic. Historically, it has been assumed that the risk of AF decreases after a non-chronic (secondary) condition has been corrected. However, there is some evidence to suggest that the risk remains. In the Framingham Heart Study, 1409 individuals with new onset AF were evaluated for their risk of subsequent occurrences based on whether they had a secondary precipitant or not [64]. A precipitant was found in 439 (31 percent) and included cardiothoracic surgery (30 percent), infection (23 percent), non-cardiothoracic surgery (20 percent), and acute myocardial infarction (18 percent). Other secondary precipitants included acute alcohol consumption, thyrotoxicosis, acute pericardial disease, acute pulmonary embolism, and other acute pulmonary pathology. While the 15-year cumulative incidence of recurrent AF was significantly lower among those with secondary causes (62 versus 71 percent), the finding that AF recurred in the majority with secondary causes was unexpected.
Surgery — AF occurs in relation to a variety of different types of surgery, with the incidence greatest in patients undergoing cardiac surgery:
●Cardiac surgery – AF has been reported in up to 30 to 40 percent of patients in the early postoperative period following coronary artery bypass graft surgery (CABG) [65-68], in 37 to 50 percent after valve surgery [65,68,69], and in as many as 60 percent undergoing valve replacement plus CABG [65,68]. This topic is discussed in detail separately. (See "Atrial fibrillation and flutter after cardiac surgery".)
●Cardiac transplantation – AF has been described in 10 to 24 percent of patients with a denervated transplanted heart, often in the absence of significant rejection [68,70]. Most episodes occur within the first two weeks, while AF developing after two weeks may be associated with an increased risk of subsequent death [70,71]. (See "Arrhythmias following cardiac transplantation", section on 'Supraventricular arrhythmias'.)
●Noncardiac surgery –AF is less common after noncardiac compared with cardiac surgery. The reported incidence of new onset AF in patients undergoing noncardiac surgery ranges from 1 and 40 percent. This broad range is likely due to variability in patient and surgical characteristics [72,73].
The largest experience comes from a review of 4181 patients over the age of 50 who were in sinus rhythm prior to major noncardiac surgery [74]. The incidence of perioperative AF was 4.1 percent; most episodes occurred within the first three days after surgery. The risk was greatest with intrathoracic surgery (odds ratio 9.2). In another series of 2588 undergoing noncardiac thoracic surgery, the incidence of AF was 12.3 percent [75].
Hyperthyroidism — Patients with hyperthyroidism have an increased risk of developing AF [76]. In one population-based study of 40,628 patients with clinical hyperthyroidism, 8.3 percent had AF or atrial flutter [77]. AF occurred in 10 to 20 percent of patients over age 60 but in less than 1 percent of patients under age 40. Men were more likely to have AF than women (12.1 versus 7.6 percent). (See "Cardiovascular effects of hyperthyroidism", section on 'Atrial fibrillation'.)
Increased beta adrenergic tone may be in part responsible for the development of AF in hyperthyroidism and may also contribute to the rapid ventricular response in this setting. In addition, excess thyroid hormone increases the likelihood of AF in experimental animals, even in the presence of beta receptor and vagal blockade [78]; it is likely that this observation applies to humans. The mechanism is unknown, but may be related to an increased automaticity and enhanced triggered activity of pulmonary vein cardiomyocytes, which can be a source of ectopic beats that initiate AF [79].
The risk of AF is also increased in patients with subclinical hyperthyroidism (defined as a low serum thyroid stimulating hormone concentration and normal serum thyroid hormone concentrations) [80-82]. The increase in risk is illustrated by the following observations:
●In a prospective study, 2007 subjects ≥60 years of age who did not have AF were followed for 10 years [80]. The subsequent age-adjusted incidence of AF was significantly higher among those with a low serum thyroid stimulating hormone concentration compared with those with a normal value (28 versus 10 per 1000 person-years).
●In a review of 23,638 subjects, the prevalence of AF in those with clinical and subclinical hyperthyroidism was similar (14 and 13 percent, respectively) and higher than that in euthyroid subjects (2.3 percent) [81].
A possible relationship between AF and hypothyroidism has been suggested but not proven [83,84]. Since hypothyroidism is present in 5 to 10 percent of the general population, it is not surprising that some patients with AF have hypothyroidism (7.7 percent with subclinical disease in one report [82]) that may not be causally related.
OTHER FACTORSA number of risk factors not discussed above are associated with an increased risk for the development of atrial fibrillation (AF).
Family history — The presence of AF in a first-degree relative, particularly a parent, has long been associated with an increase in risk, independent of standard risk factors such as age, sex, hypertension, diabetes, or clinically overt heart disease [85]. (See 'Epidemiology' above and 'Chronic disease associations' above.)
In an analysis of over 4400 individuals in the Framingham Heart Study, the occurrence of AF in a first degree relative was associated with a significantly increased risk of incident AF (multivariable-adjusted hazard ratio [HR] 1.40, 95% CI 1.13-1.74) [86]. The strength of this relationship was greater when only first-degree relatives with premature onset (age ≤65 years) were considered.
Genetic factors — For most patients with AF, one or more of the disease associations discussed above are present. In patients with lone AF, the cause(s) is unclear. In these patients, and perhaps in many of those with disease associations, a genetic basis may be present. (See "Overview of atrial fibrillation", section on 'Lone atrial fibrillation'.)
The heritability of AF is complex. For the majority of patients, genetic susceptibility, if present, is probably a polygenic phenomenon, meaning that it is due to the combined effects of a number of genes. Polygenic inheritance can explain why some diseases cluster in families, but do not demonstrate the classic Mendelian inheritance patterns of monogenic disorders. However, a small number of families demonstrate monogenic inheritance characteristics.
Polygenic inheritance — Polygenic inheritance appears to be more common and could explain the modest elevation in the relative risk of AF in first- and second-degree relatives of affected individuals. Evidence supporting a heritable component to AF susceptibility includes:
●In a review of 914 patients with AF, 50 (5 percent) had a family history of AF (one to nine additional relatives affected) [87].
●In an analysis of 2243 offspring in the Framingham Heart Study, those with parental AF had a significantly higher incidence of developing AF than those without parental AF (4.1 versus 2.7 percent, adjusted odds ratio 1.85) [85]. This effect was more pronounced when the analysis was limited to patients with age of AF onset less than 75 years and to those without prior myocardial infarction, heart failure, or valve disease (odds ratio 3.17).
●A population study in Iceland evaluated the heritability of AF in a cohort of 5269 patients diagnosed over a 16-year period [88]. Among patients with AF, the degree of relatedness was significantly greater than among matched controls. In addition, the relative risk of developing AF was higher in the relatives of patients than those of controls.
Monogenic inheritance — Some families have been identified in which AF inheritance follows more typical Mendelian patterns, consistent with a single disease-causing gene. Both autosomal dominant and autosomal recessive forms have been identified. Genetic linkage analysis has identified loci at 10q22-q24, 11p15.5, 6q14-16, 3p22-p25, and 4q25 [89-93]. At the 4q25 locus, several single-nucleotide polymorphisms have been identified [94]. In these individuals, penetrance is variable and the polymorphisms can affect the clinical expression of familial AF [95].
An autosomal recessive pattern of AF inheritance was reported in a large family from Uruguay [96]. Clinical manifestations included AF during fetal life, neonatal sudden death, ventricular tachyarrhythmias, and waxing and waning cardiomyopathy. A genetic locus on chromosome 5p13 was linked to disease in this family.
Chromosomal loci are large areas with multiple genes, and a specific genetic defect is not yet known for most loci. Examples of a monogenic cause of AF include:
●The 11p15.5 locus is associated with a gain-of-function mutation in the KVLQT1 (KCNQ1) gene, the protein product of which is the alpha-subunit of the slowly acting component of the outward-rectifying potassium current (IKs) [90]. This mutation is thought to initiate and maintain AF by reducing the action potential duration and effective refractory period in atrial myocytes.
●A different gain-of-function mutation in this gene has been associated with the congenital short QT syndrome, while loss-of-function mutations are associated with long QT syndrome, type 1. (See "Congenital long QT syndrome: Pathophysiology and genetics", section on 'Type 1 LQTS (LQT1)'.)
●An autosomal dominant form of AF, usually in association with a dilated cardiomyopathy, has been associated with mutations in SCN5A, the cardiac sodium channel gene. In a report of individuals with SCN5A mutations, 27 percent had early features of dilated cardiomyopathy (mean age at diagnosis 20 years), 43 percent had AF (mean age at diagnosis 28 years), and 38 percent had dilated cardiomyopathy (mean age at diagnosis 48 years) [92]. (See "Genetics of dilated cardiomyopathy".)
Mutations in SCN5A have also been identified in several other cardiac disorders, including the long QT syndrome, the Brugada syndrome, familial atrioventricular conduction block, and familial sinus node dysfunction. (See "Brugada syndrome: Epidemiology and pathogenesis" and "Congenital long QT syndrome: Pathophysiology and genetics" and "Etiology of atrioventricular block" and "Sinus node dysfunction: Epidemiology, etiology, and natural history".)
Birth weight — A possible relationship between birth weight and the development of AF was evaluated in a prospective study of nearly 28,000 women over the age of 45 years, who were free of AF at baseline [97]. The age-adjusted HRs (with <2.5 kg [5.5 pounds] being the referent group) for incident AF increased significantly (1, 1.3, 1.28, 1.7, and 1.71) from the lowest to the highest birth weight category (<2.5 [5.5], 2.5 [5.5] to 3.2 [7.1], 3.2 [7.1] to 3.9 [8.6], 3.9 [8.6] to 4.5, and >4.5 [9.9] kg [pounds]), during a median follow-up of 14.5 years.
Inflammation and infection — Inflammatory processes may play a role in the genesis of AF. Measurement of serum C-reactive protein (CRP), an acute phase reactant, has been used to assess the relationship between AF and inflammation.
Observational studies have reported elevated serum levels of CRP in patient populations with any of the following characteristics: later (after known high CRP) development of AF [98], history of atrial arrhythmias [99], failed cardioversion [100], recurrence of AF after cardioversion [101], and development of AF after cardiac surgery. (See "Atrial fibrillation and flutter after cardiac surgery".)
However, inflammation is more likely a marker for other conditions associated with AF, as opposed to being a direct cause or a perpetuating agent. The strongest evidence against a direct causal role for inflammation, as detected by an elevation in CRP, comes from a Mendelian randomization study that evaluated nearly 47,000 individuals in two cohorts from Copenhagen, Demark [102]. (See "Mendelian randomization".)
The following observations were made:
●After multifactorial adjustment, a CRP level in the upper versus lower quintile was associated with a significantly increased risk of the development of AF (hazard ratio 1.77, 95% CI 1.22-2.55).
●Genotype combinations of four CRP polymorphisms were significantly associated with up to a 63 percent increase in plasma CRP levels, but not with an increased risk of the development of AF.
Thus, inflammation, as determined by CRP, is not likely to be causative of AF. In addition to inflammation as detected by serum CRP, new onset AF and other acute cardiac events have been associated with pneumococcal pneumonia [103]. (See "Pneumococcal pneumonia in adults", section on 'Acute cardiac events'.)
The risk of AF increases after infection with the influenza virus [104].
Pericardial (epicardial) fat — Pericardial fat, also referred to as epicardial fat, is the fat depot within the pericardial sac, which is in between the visceral and parietal pericardium. It has inflammatory properties: both obesity and inflammation are risk factors for AF [105,106]. In a study of 126 patients with AF and 76 controls, those with AF had a significantly higher pericardial fat volume (102 versus 76 ml) [105]. This was true for patients with either paroxysmal or persistent AF, and was independent of other traditional predictors of AF, including left atrial enlargement.
Autonomic dysfunction — The autonomic nervous system may be involved in the initiation and maintenance of AF. It may be particularly important in patients with paroxysmal AF, as both heightened vagal and sympathetic tone can promote AF. Vagal tone is predominant in normal hearts, which may explain why vagally-mediated AF is often seen in athletic young men without apparent heart disease who have slow heart rates during rest or sleep; such patients may also have an electrocardiogram (ECG) pattern of typical atrial flutter alternating with AF [107,108]. In comparison, AF induced by increased sympathetic tone may be observed in patients with underlying heart disease or during exercise or other activity [108]. (See "Paroxysmal atrial fibrillation", section on 'Pathogenesis'.)
Findings on the electrocardiogram — Abnormal QT or P-wave duration are associated with an increased risk of AF:
●Corrected QT interval – Individuals with either congenital long QT syndrome or short QT syndrome have an increased risk of AF [109,110]. (See "Congenital long QT syndrome: Epidemiology and clinical manifestations" and "Short QT syndrome".)
The issue of whether there is an association between the corrected QT interval (QTc) and the risk of AF in the general population was addressed in a study of 281,277 individuals without baseline AF in the greater Copenhagen region who were followed for a median period of 5.7 years after a first ECG [111]. Individuals with a QTc <372 ms (1st percentile) or ≥419 ms (60th percentile) had an increased risk (adjusted hazard ratios 1.45 up to 1.44, respectively) compared with the reference group (411 to 419 ms). The risk increased in a dose-dependent manner above 419 ms and was strongest among individuals with lone AF.
●P-wave duration – The relationship between P-wave duration and the risk of AF was evaluated in a study of nearly 285,933 individuals, of whom 9550 developed AF during a median follow-up period of 6.7 years [112]. Compared with the reference group (100 to 105 milliseconds [ms]), individuals with very short (≤89 ms; HR 1.6), intermediate (112 to 119 ms; HR 1.22), long (120 to 129; HR 1.5), and very long P wave duration (≥130 ms; HR 2.06) had an increased risk.
Premature atrial contractions — Premature atrial contractions (PACs) are important as a trigger of PAF. (See 'Pathogenesis' above.) The issue of whether they are predictor of incident AF was evaluated in a study of 1260 adults without prevalent AF who underwent 24-hour ambulatory ECG (Holter) monitoring at baseline [113]. During a median follow-up of 13.0 years, 27 percent developed incident AF. After adjusting for other known predictors of AF, PAC count (by quartile) was significantly associated with incident AF; the adjusted HRs comparing quartile 2, 3, and 4 with quartile 1 were 2.17, 2.79, and 4.92, respectively.
Other supraventricular tachyarrhythmias — Spontaneous transition between typical atrial flutter and AF has been observed, although little is known about the mechanism of this conversion [114,115]. In addition, AF is, in some patients, associated with paroxysmal supraventricular tachycardia (PSVT) [116-118]. The most common causes of PSVT are atrioventricular nodal re-entrant tachycardia and atrioventricular reentrant tachycardia, which occurs in patients with the Wolff-Parkinson-White syndrome or concealed accessory pathways. (See "Atrioventricular nodal reentrant tachycardia" and "Atrioventricular reentrant tachycardia (AVRT) associated with an accessory pathway".)
The association between AF and PSVT was illustrated in a report that evaluated 169 patients who presented with PSVT and were followed by clinic visits and transtelephonic ECG monitoring during symptomatic episodes [116]. Nineteen percent had an episode of AF during a mean follow-up of 31 months. Neither the mechanism nor the rate of the PSVT was associated with the time to occurrence of AF.
Enhanced vagal tone, as determined by baroreflex sensitivity, or an increase in dispersion of right atrial refractory periods may contribute to the development of AF associated with PSVT [119]. Alternatively, an atrial premature beat can cause stable PSVT to degenerate into AF.
Among patients with Wolff-Parkinson-White syndrome, the mechanism of AF may be retrograde conduction via the accessory pathway of a premature beat, stimulating the atrial myocardium during its vulnerable period [120]. Ablation of the accessory pathway reduces the incidence of subsequent AF [120,121].
Low serum magnesium — In an observational study of over 3500 participants in the Framingham Offspring Study, individuals in the lower quartile of serum magnesium were approximately 50 percent more likely to develop AF compared with those in the upper quartiles after multivariable adjustment [122].
Alcohol — AF occurs in up to 60 percent of binge drinkers with or without an underlying alcoholic cardiomyopathy [123]. Most cases occur during and following weekends or holidays when alcohol intake is increased, a phenomenon that has been termed "the holiday heart syndrome." However, even modest amounts of alcohol can trigger AF in some patients. (See "Alcoholic cardiomyopathy".)
Moderate, long-term alcohol consumption was not shown to be a risk factor for AF in relatively small studies [58,124,125]. However, a positive association was found in a 2014 study of 79,019 men and women free from AF at baseline [126]. Compared with current drinkers of <1 drink per week, the multivariable risk ratios of AF were 1.01 (95% CI 0.94-1.09) for one to six drinks per week, 1.07 (95% CI 0.98-1.17) for 7 to 14 drinks per week, 1.14 (95% CI 1.01-1.28) for 15 to 21 drinks per week, and 1.39 (95% CI 1.22-1.58) for >21 drinks per week.
In contrast, heavy alcohol consumption is associated with an increased incidence of AF. Two large cohort studies found an increased incidence among men with heavy alcohol consumption (HR 1.45 in both) [127,128]. Neither study found a correlation between heavy alcohol use and AF in women, but the ability to detect such a correlation was limited by the small numbers of women with alcohol consumption in this range. Another study of 1055 cases of AF occurring during long-term follow-up found an increased risk (relative risk 1.34, 95% CI 1.01-1.78) with consumption of more than 36 grams per day (approximately >3 drinks/day) [125].
Caffeine — There is a widespread belief that caffeine, particularly at high doses, is associated with palpitations and a number of arrhythmias, including AF and supraventricular and ventricular ectopy. However, despite the theoretical relationship between caffeine and arrhythmogenesis, there is no evidence in humans that ingestion of caffeine in doses typically consumed can provoke AF or any other spontaneous arrhythmia [129]. (See "Cardiovascular effects of caffeine and caffeinated beverages", section on 'Arrhythmias'.)
Fish and fish oil supplements — Observational data has suggested that dietary fish intake or fish oil supplements, particularly those rich in long-chain n-3 fatty acids, may reduce the incidence of arrhythmias, although evidence is mixed with regard to both atrial and ventricular arrhythmias. (See "Overview of sudden cardiac arrest and sudden cardiac death", section on 'Fish intake and fish oil'.)
With regard to dietary fish intake and incident AF, three cohort studies (approximately 45,000, 48,000, and 5000 individuals) found no relationship [130-132], while one (approximately 5000 individuals) suggested a reduction in AF burden [133].
Medications — Certain medications can cause or contribute to the development of AF [134]. These include theophylline [135], adenosine [136], and, since increased vagal tone can induce AF [108], drugs that enhance vagal tone, such as digitalis. (See 'Autonomic dysfunction' above.)
Bisphosphonates (eg, alendronate, risedronate, etidronate) are widely used in the treatment of osteoporosis, and concern has been raised that these drugs can cause AF. The weight of evidence suggests that the risk of AF from oral bisphosphonates is small, if it exists at all. (See "Risks of bisphosphonate therapy in patients with osteoporosis", section on 'Atrial fibrillation'.)
Case-control studies have suggested a modest increased risk for the development of AF in patients taking nonsteroidal anti-inflammatory drugs [137-139]. However, the absence of an accepted biologic mechanism and the susceptibility of case-control studies to unmeasured confounders makes us cautious about the strength of this association [140].
Regular physical activity — Some [141,142], but not all [143,144], studies have suggested a positive association between regular physical activity and the risk of AF in the general population. In a 2013 meta-analysis of four prospective cohorts (n = 43,672), and after dividing subjects into four or five groups on the basis of cumulative physical activity per week, there was no difference in the risk of AF comparing patients in the maximum and minimal groups (odds ratio 1.08, 95% CI 0.97-1.21) [145].
Air pollution — Air pollution, and specifically fine particulate matter, is associated with increased cardiovascular disease mortality. (See "Overview of possible risk factors for cardiovascular disease", section on 'Air pollution'.)
Whether air pollution is associated with episodes of AF was evaluated in a study of 176 patients with dual chamber implantable cardioverter-defibrillators that were capable of detecting episodes of AF. After follow-up of nearly two years, there were 328 episodes of AF lasting 30 seconds or more found in 49 patients [146]. The potential impact of multiple parameters of air pollution, (measured hourly) on the development of AF was examined. The odds of AF increased significantly as the concentration of particulate matter increased in the two hours prior to the event.
RISK PREDICTION MODELSModels that attempt to predict the risk of development of atrial fibrillation (AF) have been developed but are not widely employed.
Using the Framingham Heart Study population, age, sex, systolic blood pressure, treatment for hypertension, PR interval, clinically significant cardiac murmur, body mass index, and heart failure were incorporated into a risk prediction model that predicts an individual's absolute risk over 10 years [147]. This model has been validated in two geographically and racially diverse cohorts in the age range of 45 to 95 years and predicted the five-year incidence of AF with moderate accuracy (C statistic 0.66 to 0.68) [148]. Other models have been studied, including one in a racially diverse population [149].
However, the benefit of using risk prediction models in this setting has not been established. There are no studies linking this with improved outcomes.
SUMMARY
●The incidence and prevalence of atrial fibrillation (AF) depends upon the population studied and the intensity of monitoring. Both increase significantly with increasing age. (See 'Epidemiology' above.)
●Hypertensive heart disease and coronary heart disease are the most common chronic disease associations in patients with AF in developed countries. Other frequent causes include alcohol excess, heart failure, valvular heart disease including both regurgitant and stenotic lesions, and hyperthyroidism. (See 'Chronic disease associations' above.)
●AF occurs in relation to a variety of different types of surgery; the incidence is greatest in patients undergoing coronary artery bypass graft or cardiac valve surgery. (See 'Chronic disease associations' above.)
●Chronic, heavy alcohol use does increase the risk of AF in men, while the impact of heavy alcohol use in women is less clear. Chronic moderate alcohol use does not appear to increase the incidence of AF in men or women. (See 'Chronic disease associations' above.)
●The heritability of AF is complex. For the majority of patients, genetic susceptibility, if present, is probably a polygenic phenomenon, meaning that it is due to the combined effects of a number of genes. (See 'Genetic factors' above.)
●Models to predict the risk of subsequent AF have been developed. However, a benefit from using such models has not been established. (See 'Risk prediction models' above.)
+++++++++++++
Atrial fibrillation can lead to blood clots forming in the heart that may circulate to other organs and lead to blocked blood flow (ischemia).
On ECG, AF looks like an irregularly irregular, usually narrow-complex, tachycardia without associated P-waves.
The RR intervals follow no repetitive pattern. They have been labeled as “irregularly irregular.”
While electrical activity suggestive of P waves is seen in some leads, there are no distinct P waves. Thus, even when an atrial cycle length (the interval between two atrial activations or the P-P interval) can be defined, it is not regular and often less than 200 milliseconds (translating to an atrial rate greater than 300 beats per minute).
Atrial fibrillation with an irregularly irregular rhythm with no p-waves. Note the fibrillatory or f waves.
AF affects 1% of the general population.
The prevalence of AF increases with age.
AF is more common in men than in women and more common in whites than in blacks.
The two goals of therapy are symptom control and the prevention of thromboembolism.
1. Symptom Control
Rhythm- and rate-control strategies are associated with similar rates of mortality and serious morbidity, such as embolic risk. In addition, assessments of quality of life have not shown significant differences between the two in most studies.
The findings suggest that rate-control therapy alone can achieve prompt symptom relief in almost all eligible patients, with good quality of life and a low risk of complications, while facilitating rapid discharge from the emergency department. 1
A rate-control strategy generally uses drugs that block the atrioventricular (AV) node, such as beta blockers, rate-slowing calcium channel blockers, or digoxin. AV nodal ablation plus ventricular pacing is another option.
The principal reasons to prefer a rate-control strategy include simplification of the medical regimen, lower cost, and less concern about the risks of antiarrhythmic drug therapy (such as torsades de pointes) or radiofrequency catheter ablation. (See 'Preference for rate control' above.)
●Patients managed by either rate or rhythm control must be assessed for their thromboembolic risk. Therapy should be initiated when appropriate. (See 'Thromboembolic risk' above.)
●For asymptomatic or mildly symptomatic AF patients who are 65 years or older, we suggest a rate-control as opposed to a rhythm-control strategy using medical therapy (Grade 2B). This recommendation places a high priority on concerns about side effects of antiarrhythmic drug therapy or radiofrequency catheter ablation. Patients for whom a rhythm-control strategy may be reasonable include those who continue with clinically significant symptoms on a rate-control strategy. (See 'Preference for rate control' above.)
started on sustained-release metoprolol, 50 mg/d.
After 7 days, her rhythm reverts to normal sinus rhythm spontaneously. However, over the ensuing month, she continues to have intermittent palpitations and fatigue. Continuous ECG recording over a 48-hour period documents paroxysms of atrial fibrillation with heart rates of 88–114 bpm. An echocardiogram shows a left ventricular ejection fraction of 38% (normal ≥ 60%) with no localized wall motion abnormality. At this stage, would you initiate treatment with an antiarrhythmic drug to maintain normal sinus rhythm, and if so, what drug would you choose?
Cardiac arrhythmias are a common problem in clinical practice, occurring in up to 25% of patients treated with digitalis, 50% of anesthetized patients, and over 80% of patients with acute myocardial infarction*. Arrhythmias may require treatment because rhythms that are too rapid, too slow, or asynchronous can reduce cardiac output. Some arrhythmias can precipitate more serious or even lethal rhythm disturbances; for example, early premature ventricular depolarizations can precipitate ventricular fibrillation. In such patients, antiarrhythmic drugs may be lifesaving. On the other hand, the hazards of antiarrhythmic drugs—and in particular the fact that they can precipitate lethal arrhythmias in some patients—have led to a reevaluation of their relative risks and benefits. In general, treatment of asymptomatic or minimally symptomatic arrhythmias should be avoided for this reason.
Arrhythmias can be treated with the drugs discussed in this chapter and with nonpharmacologic therapies such as pacemakers, cardioversion, catheter ablation, and surgery. This chapter describes the pharmacology of drugs that suppress arrhythmias by a direct action on the cardiac cell membrane. Other modes of therapy are discussed briefly (see Box: The Nonpharmacologic Therapy of Cardiac Arrhythmias, later in the chapter).
Prevention of Thromboembolism
Use anticoagulation based on the CHADS2 or CHA2DS2-VASc criteria. (See 'Thromboembolic risk' above.)
She is anticoagulated with warfarin and
●For most patients with AF younger than age 65, particularly those who are symptomatic, we suggest a rhythm control as opposed to a rate-control strategy (Grade 2B). This recommendation places a high priority on relief of symptoms as well as the potential, but as of yet unproven benefit, from remaining in sinus rhythm over long periods of time. For younger, asymptomatic patients who are concerned about the potential side effects of antiarrhythmic drug therapy, and who are not inclined to undergo radiofrequency catheter ablation, a rate-control strategy is reasonable. In these patients, an informed discussion of the benefits and risks of antiarrhythmic drug therapy is critically important. (See 'Preference for rhythm control' above.)
[Efforts to treat with ablation or devices show promise, but to date, pharmacologic therapy represents the mainstay of AF treatment.]
The benefit of efforts to control the rhythm (as opposed to rate control with anticoagulation) is debated, and mortality benefit may be clarified by the ongoing AFFIRM trial; nevertheless, most physicians remain committed to reducing symptoms through restoration and maintenance of sinus mechanism.
Generally, the agents employed to stabilize the atria are the drugs in the Vaughan Williams categories of class IA, IC and III. In this issue of the Journal, Kühlkamp et al. (1) present data that show beta-adrenergic blockade also acts to reduce the frequency of symptomatic AF.
1. Search for treatable contributing factors such as???
2. Control the heart rate (How???
[Studies have shown that rhythm control and rate control result in similar mortality and stroke rates, even in patients with underlying heart failure. Quality of life should be considered when deciding upon treatment}
Therapy is directed at rate control with a goal of less than 100 bpm. Rate control is generally achieved with
beta-blockers, (dose?)
a calcium channel blocker, (dose?)
or digoxin. (dose?)
Digoxin is less effective in controlling ventricular response during activity and in paroxysmal atrial fibrillation. It is useful in patients with decreased LV function.
Preventing thromboembolism. (dose?)
Anticoagulation should be started in patients at high risk for stroke as determined by the CHADS2 scoring system. (See scoring system below.)
CHADS2 system:
Treatment is guided by patients’ symptoms, the hemodynamic effect of AF, the duration of AF if there are persistent risk factors for stroke, and underlying heart disease.
In a patient with symptomatic AF with RVR, an early priority in management will be to slow the ventricular rate.
For most patients with AF, the typical symptoms of palpitations and dyspnea can be alleviated through simple rate control. In rare cases, tachycardia and loss of the “atrial kick” can lead to diminished CO, hypotension, or congestive heart failure. In those cases, if the arrhythmia is thought to be the primary cause of the patient’s instability, emergent electrical cardioversion is indicated. In more stable patients, the decision of whether to convert the rhythm depends on a number of factors, including risk of thromboembolism, need for anticoagulation, and odds of recurrent AF. In all patients, a search for the underlying etiology should be undertaken because AF is often best managed by treating the underlying cause of the rhythm rather than the rhythm itself (Table 9–1).
In this patient with symptomatic AF with RVR, an early priority in management will be to slow the ventricular rate. (How?)
For most patients with AF, the typical symptoms of palpitations and dyspnea can be alleviated through simple rate control. In rare cases, tachycardia and loss of the “atrial kick” can lead to diminished CO, hypotension, or congestive heart failure. In those cases, if the arrhythmia is thought to be the primary cause of the patient’s instability, emergent electrical cardioversion is indicated. In more stable patients, the decision of whether to convert the rhythm depends on a number of factors, including risk of thromboembolism, need for anticoagulation, and odds of recurrent AF. In all patients, a search for the underlying etiology should be undertaken because AF is often best managed by treating the underlying cause of the rhythm rather than the rhythm itself.
Cardiac | Hypertension (approximately 80% of cases), coronary artery disease, cardiomyopathy, valvular heart disease, rheumatic heart disease, congenital heart disease, myocardial infarction, pericarditis, myocarditis |
Pulmonary | Pulmonary embolism, chronic obstructive pulmonary disease (COPD), obstructive sleep apnea |
Systemic disease | Hyperthyroidism, obesity, metabolic syndrome, inflammation |
Postoperative | Cardiac surgery, any surgery |
Binge alcohol drinking | “Holiday heart syndrome” |
Lone AF | Associated with approximately 10% of AF |
{
Oral anticoagulation in high-risk patients with AF includes vitamin K antagonists or
the newer anticoagulants such as thrombin inhibitors (dabigatran) or
factor Xa inhibitors (rivaroxaban, apixaban),
but not antiplatelet agents (aspirin and clopidogrel), which have substantially less effect.
Electric Cardioversion
New-onset AF that produces severe hypotension, pulmonary edema, or angina should be electrically cardioverted starting with a QRS synchronous shock of 200 J, ideally after sedation or anesthesia is achieved. Greater shock energy and different electrode placements may be tried if the shock fails to terminate AF.
If AF terminates and reinitiates, administration of an antiarrhythmic drug, such as ibutilide, and repeat cardioversion may be considered. If the patient is stable, immediate management involves rate control to alleviate or prevent symptoms, anticoagulation if appropriate, and cardioversion to restore sinus rhythm if AF is persistent.
Anticoagulation strategies for new-onset AF are debated. In the absence of contraindications, it is usually appropriate to initiate systemic anticoagulation with heparin immediately, while evaluation and other therapies are implemented.
CARDIOVERSION AND ANTICOAGULATION
Cardioversion within 48 h of the onset of AF is common practice in patients who have not been anticoagulated, provided that they are not at high risk for stroke due to a prior history of embolic events, rheumatic mitral stenosis, or hypertrophic cardiomyopathy with marked left atrial enlargement. These patients are usually at risk of recurrence, such that initiation of anticoagulation is considered based on the patient’s individual risk for stroke, commonly assessed from the CHA2DS2-VASc score.
If the duration of AF exceeds 48 h or is unknown, there is greater concern for thromboembolism with cardioversion, even in patients considered low risk for stroke. There are two approaches to mitigate the risk related to cardioversion. One option is to anticoagulate continuously for 3 weeks before and a minimum of 4 weeks after cardioversion. A second approach is to start anticoagulation and perform a transesophageal echocardiogram to determine if thrombus is present in the left atrial appendage. If thrombus is absent, cardioversion can be performed and anticoagulation continued for aminimum of 4 weeks because recovery of atrial mechanical function after electrical or pharmacologic cardioversion may be delayed and thrombus can form and embolize days after cardioversion. Some patients may merit ongoing anticoagulation after cardioversion, depending on stroke risk profile.
RATE CONTROLAcute rate control can be achieved with beta blockers and/or the calcium channel blockers verapamil and diltiazem administered either intravenously or orally, as warranted by the urgency of the clinical situation. Digoxinmay be added, particularly in heart failure patients, because it does not have negative inotropic effects, particularly if use of AV nodal–blocking agents is limited by poor tolerance or is contraindicated. Its effect is modest but synergistic with the other AV nodal–blocking agents, but it is particularly limited when sympathetic tone is elevated. Typically, the goal of acute rate control is to reduce the ventricular rate to less than 100/min, but the goal must be guided by the clinical situation.
CHRONIC RATE CONTROLFor patients who remain in AF chronically, the goal of rate control is to alleviate and prevent symptoms and prevent deterioration of ventricular function from excessive rates. β-Adrenergic blockers, calcium channel blockers, and digoxin are used, sometimes in combination. Rate should be assessed with exertion and medications adjusted accordingly. Exertion-related symptoms are often an indication of inadequate rate control. The initial goal is a resting heart rate of less than 80 beats/min that increases to less than 100 beats/min with light exertion, such as walking. If it is difficult to slow the ventricular rate to that degree, allowing a resting rate of up to 110 beats/min is acceptable provided it does not cause symptoms and ventricular function remains normal. Periodic assessment of ventricular function is warranted because some patients develop tachycardia-induced cardiomyopathy.
If adequate rate control in AF is difficult to achieve, further consideration should be given to restoring sinus rhythm. Catheter ablation of the AV junction to create heart block and implantation of a permanent pacemaker reliably achieve rate control without the need for AV nodal agents, but implement life-long permanent pacing. Right ventricular apical pacing induces dyssynchronous ventricular activation that can be symptomatic or depress ventricular function in some patients. Biventricular pacing may be used to minimize the degree of ventricular dyssynchrony.
STROKE PREVENTION IN ATRIAL FIBRILLATIONThe majority of patients warrant chronic anticoagulation, but selection of therapy should be individualized based on patient profile and risks and benefits of individual agents. Anticoagulation with a vitamin K antagonist is warranted for all patients with AF who have rheumatic mitral stenosis or mechanical heart valves for whom the newer anticoagulants have not been tested. Anticoagulation with a vitamin K antagonist (warfarin) or the newer oral anticoagulants is warranted for patients who have had more than 48 h of AF and are undergoing cardioversion, for patients who have a prior history of stroke, or for patients with a CHA2DS2-VASc score of ≥2, but it may be considered in patients with a risk score of 1. The approach to patients with paroxysmal AF is the same as for persistent AF. It is recognized that many patients who appear to have infrequent AF episodes often have asymptomatic episodes that put them at risk. Absence of AF during periodic monitoring is not sufficient to indicate low risk. The role of continuous monitoring with implanted recorders or pacemakers is not yet clear as a guide for anticoagulation in patients with a borderline risk profile. Bleeding is the major risk of anticoagulation. Major bleeding requiring transfusion or in a critical area (e.g., intracranial) occurs in approximately 1% of patients per year. Risk factors for bleeding include age >65–75 years, heart failure, history of anemia, and excessive alcohol or nonsteroidal anti-inflammatory drug use. Patients with coronary stents who require antiplatelet therapy with aspirin and a thienopyridine are at particularly high risk of bleeding.
Warfarin reduces the annual risk of stroke by 64% compared to placebo and by 37% compared to antiplatelet therapy. The newer anticoagulants, dabigatran, rivaroxaban, and apixaban, have been found to be noninferior to warfarin in individual trials, and analysis of pooled data suggests superiority to warfarin by small absolute margins of 0.4–0.7% in reduction of mortality, stroke, major bleeding, and intracranial hemorrhage. Warfarin is an inconvenient agent that requires several days to achieve a therapeutic effect (prothrombin time [PT]/international normalized ratio [INR] >2), requires monitoring of PT/INR to adjust dose, and has many drug and food interactions, thus limiting patient compliance. The newer agents are easier to use and achieve reliable anticoagulation promptly without requiring dosage adjustment based on blood tests. Dabigatran, rivaroxaban, and apixaban have renal excretion, cannot be used with severe renal insufficiency, and require dose adjustment for modest renal impairment, which is of particular concern in the elderly, who are at increased bleeding risk. Excretion can also be influenced by P-glycoprotein inducers and inhibitors. Warfarin anticoagulation can be reversed by administration of fresh frozen plasma and vitamin K. Reversing agents for the newer anticoagulants are lacking (but in development), and bleeding must be managed with supportive care, with the expectation that clotting will improve over 12 h as the anticoagulant is excreted.
The antiplatelet agents aspirin and clopidogrel are inferior to warfarin for stroke prevention in AF and do not reduce the risk of bleeding. Clopidogrel combined with aspirin is better than aspirin alone but inferior to warfarin and has greater bleeding risk than aspirin alone.
Chronic anticoagulation is contraindicated in some patients due to bleeding risks. Because most atrial thrombi are felt to originate in the left atrial appendage, surgical removal of the appendage, combined with atrial maze surgery, may be considered for patients undergoing surgery, although removal of the appendage has not been unequivocally shown to reduce the risk of thromboembolism. Percutaneous devices that occlude or ligate the left atrial appendage are being studied for safety and efficacy.
RHYTHM CONTROLThe decision to administer antiarrhythmic drugs or perform catheter ablation to attempt maintenance of sinus rhythm (commonly referred to as the “rhythm control strategy”) is mainly guided by patient symptoms and preferences regarding the benefits and risks of therapies. In general, patients who maintain sinus rhythm have better survival than those who continue to have AF. This is likely because continued AF is a marker of disease severity. In randomized trials, administration of antiarrhythmic medications to maintain sinus rhythm did not improve survival or symptoms compared to a rate control strategy, and the drug therapy group had more hospitalizations. Disappointing efficacy and toxicities of available antiarrhythmic drugs and patient selection bias may be factors that influenced the results of these trials. The impact of catheter ablation on mortality is not known. A rhythm control strategy is usually selected for patients with symptomatic paroxysmal AF, a first episode of symptomatic persistent AF, AF with difficult rate control, and AF that has resulted in depressed ventricular function or that aggravates heart failure. A rhythm control strategy is more likely to be favored in younger patients than in sedentary or elderly patients in whom rate control is usually easily achieved. Even if sinus rhythm is apparently maintained, anticoagulation is recommended according to the CHA2DS2-VASc stroke risk profile because asymptomatic episodes of AF are common. Following a first episode of persistent AF, a strategy using AV nodal–blocking agents, cardioversion, and anticoagulation is reasonable, in addition to addressing possible aggravating factors, including hypertension, heart failure, and sleep apnea. If recurrences are infrequent, periodic cardioversion is reasonable.
Pharmacologic Therapy for Maintaining Sinus RhythmThe goal of pharmacologic therapy is to maintain sinus rhythm or reduce episodes of AF. Drug therapy can be instituted once sinus rhythm has been established or in anticipation of cardioversion. β-Adrenergic blockers and calcium channel blockers help control ventricular rate, improve symptoms, and possess a low-risk profile, but have low efficacy for preventing AF episodes. Risks and side effects of antiarrhythmic drugs are a major consideration in selecting therapy. Class I sodium channel–blocking agents (e.g., flecainide, propafenone, disopyramide) are options for subjects without significant structural heart disease, but they have negative inotropic and proarrhythmic effects that warrant avoidance in patients with coronary artery disease or heart failure. The class III agents sotalol and dofetilide can be administered to patients with coronary artery disease or structural heart disease but have approximately a 3% risk of inducing excessive QT prolongation and torsades des pointes. Dofetilide should be initiated only in a hospital with ECG monitoring, and many physicians take this approach with sotalol as well. Dronedarone increases mortality in patients with heart failure. All of these agents have modest efficacy in patients with paroxysmal AF, of whom approximately 30–50% will benefit. Amiodarone is more effective, maintaining sinus rhythm in approximately two-thirds of patients. It can be administered to patients with heart failure and coronary artery disease. Over 20% of patients experience toxicities during long-term therapy.
CATHETER AND SURGICAL ABLATION FOR ATRIAL FIBRILLATIONCatheter ablation avoids antiarrhythmic drug toxicities but has procedural risks and requires an experienced center. For patients with previously untreated but recurrent paroxysmal AF, catheter ablation has similar efficacy to antiarrhythmic drug therapy and is superior to antiarrhythmic drugs for patients who have recurrent AF despite drug treatment. The procedure involves cardiac catheterization, transatrial septal puncture, and radiofrequency ablation or cryoablation to electrically isolate the regions around the pulmonary veins, abolishing the effect of triggering foci to interact with the left atrial AF substrate. Extensive areas of ablation are required, and gaps in healed ablation areas necessitate a repeat procedure in 20–50% of patients. Sinus rhythm is maintained for more than 1 year after one procedure in approximately 60% of patients and in 70–80% of patients after multiple procedures. Some patients become more responsive to antiarrhythmic drugs.
There is a 2–7% risk of major complications, including stroke (0.5–1%), cardiac tamponade (1%), phrenic nerve paralysis, bleeding from femoral access sites, and fluid overload with heart failure, that can emerge 1–3 days after the procedure. It is important to recognize the potential for delayed presentation of some complications. Ablation within the pulmonary veins can lead to pulmonary vein stenosis, presenting weeks to months after the procedure with dyspnea or hemoptysis. Esophageal ulcers can form immediately after the procedure and may rarely lead to a fistula between the left atrium and esophagus (estimated incidence of 0.1%) that presents as endocarditis and stroke 10 days to 3 weeks after the procedure.
Catheter ablation is less effective for persistent AF. More extensive ablation is often required, including areas that likely support reentry in regions outside the pulmonary venous antra, but individual strategies are debated. More than one ablation procedure is often required to maintain sinus rhythm.
Surgical ablation of AF is typically performed concomitant with cardiac valve or coronary artery surgery and less commonly as a stand-alone procedure; however, for patients with persistent AF, surgical or hybrid procedures may have higher single-procedure efficacy. Risks include sinus node injury requiring pacemaker implantation. Surgical removal of the left atrial appendage may reduce stroke risk, although thrombus can form in the remnant of the appendage or if the appendage is not completely ligated.
Early—diminished cardiac output (CO) and hypotension.
Late—thromboembolism (stroke) and cardiomyopathy.
Content 3
Content 13
A 73-year-old woman presents to the emergency department complaining of palpitations and new shortness of breath with minor exertion for the past 2 weeks. Previously, she could walk everywhere, but now she becomes fatigued climbing the stairs of her home. Occasionally, she has felt her heart racing even when she is at rest. Her past medical history is notable for diet controlled diabetes and hypertension, for which she takes hydrochlorothiazide and amlodipine. On physical examination, she appears comfortable and speaks in full sentences without difficulty. Her blood pressure is 130/90 mm Hg, heart rate is 144 beats per minute, respiratory rate is 18 breaths per minute, oxygen saturation is 98% on room air, and temperature is 37°C (98.6°F). The head and neck examination is unremarkable. Her lungs are clear to auscultation. Her heartbeat is irregular and rapid, without murmurs, rubs, or gallops. She has no extremity edema or jugular venous distension. Her abdomen is soft and nontender, without masses. Labs show a normal complete blood count (CBC), normal electrolytes, blood urea nitrogen (BUN), creatinine, troponin, brain natriuretic peptide (BNP), and thyroid stimulating hormone. A chest x-ray reveals a normal cardiac silhouette with no pulmonary edema. The ECG is shown below.
Electrocardiogram. (Reproduced with permission from Tintinalli JE, Kelen GD, Stapczynski JS. Emergency Medicine. 6th ed. New York, NY: McGraw-Hill Education; 2004:185.)
What is your diagnosis?
Atrial fibrillation (AF) with rapid ventricular response (RVR).
A 79-year-old man with a history of coronary artery disease, ischemic cardiomyopathy with a last left ventricular (LV) ejection fraction of 30%, and hypertension presents to your office with no new complaints. Blood pressure is 108/56 mmHg, heart rate is 88 bpm, and arterial oxygen saturation is 98%. His rhythm strip is shown in Figure V-36. Based on this ECG, the patient now has a definite (class I) indication for which of the following therapies?
FIGURE V-36
Amiodarone 400 mg daily
Aspirin 325 mg daily
Flecainide 600 mg PRN palpitations
Systemic anticoagulation with warfarin or a novel oral anticoagulant
Transesophageal echocardiography followed by direct current (DC) cardioversion
The answer is D. This patient has new atrial fibrillation. When one assesses a patient with new atrial fibrillation, it is prudent to proceed through a series of decisions systematically. If the patient is hemodynamically unstable (low blood pressure, pulmonary edema, poor mentation, low urine output), then urgent direct current cardioversion is warranted. This patient is clearly stable. The next decision hinges around rate versus rhythm control. Several studies have shown clinical equipoise between rate and rhythm control strategies, with patients randomized to the rhythm control strategies undergoing far more procedures and taking more medications than patients randomized to the rate control strategies. This is also true in heart failure patients. Given the lack of symptoms and already controlled resting heart rate, rhythm control with either medications (amiodarone) or cardioversion is not a class I indication. In fact, a type I antiarrhythmic such as flecainide would be contraindicated for this patient because it has been shown to increase mortality in patients with coronary disease. The final question revolves around anticoagulation. Patients with atrial fibrillation have varying degrees of risk of thromboembolic events. Patients are stratified into risk categories by assessing set risk factors. One can remember the CHADS2 mnemonic to recall each point of the risk scoring system (congestive heart failure [CHF], hypertension, age >75, diabetes, and stroke/cerebrovascular accident [which receives 2 points]). A more sensitive score includes vascular disease, age >65, and female sex (CHADS-VASc). Any combined score >1 warrants systemic anticoagulation. Patients with a score of 0 do not require systemic anticoagulation and can take full-strength aspirin. A score of 1 is intermediate and requires an in-depth discussion with the patient about their risk threshold for anticoagulation. Many experts recommend systemic anticoagulation with a CHADS2score of 1. This patient has a CHADS2 score of 3 (CHF, hypertension, age) and thus warrants systemic anticoagulation.
An 85-year-old woman with no prior cardiac history presents to the emergency department with 2 hours of palpitations. Blood pressure, oxygen saturation, and heart rate are normal, although you note an irregularly irregular rhythm on examination. ECG shows an irregularly irregular, narrow QRS without discernable P waves at a rate of 75 bpm. Echocardiogram reveals no structural heart disease. Despite the normal heart rate, the patient is quite symptomatic with her atrial fibrillation and wants to pursue achievement of sinus rhythm. All of the following interventions may be beneficial EXCEPT:
Adenosine intravenously
Amiodarone intravenously
Direct current cardioversion
Dofetilide orally
Flecainide orally
The correct answer is A. You answered C.
The answer is A.(Chap. 276) Adenosine is a powerful “nodal-blocking agent”; that is, it blocks propagation of electrical signal through the AV node. It has a very short half-life. Administration of adenosine to a patient in atrial fibrillation will induce a transient complete heart block but will not stop the atrial fibrillation. When the adenosine wears off in a few seconds and AV conduction resumes, the ventricular rhythm will again be irregularly irregular. All the other choices have been shown to achieve sinus rhythm better than placebo in patients with atrial fibrillation.
A 75-year-old man is found to have asymptomatic AF. Which of the following is the most common complication of his AF long term?
Myocardial infarction
Stroke
Hypotension
Cardiomyopathy
The correct answer is B.
B. The two complications associated with AF are stroke and cardiomyopathy. Stroke is two to three times more likely in patients with AF than in the general population. While cardiomyopathy is also a complication of AF, it is far less common than thromboembolism.
A 65-year-old man with a history of atrial fibrillation, chronic kidney disease, and congestive heart failure is hospitalized for an exacerbation of congestive heart failure. He is maintained on warfarin with a target international normalized ratio of 2.0 to 3.0. His liver is palpable 2 cm below the right costal margin. There is 2+ pitting pretibial and ankle edema bilaterally.
His current international normalized ratio is 11. There is no evidence of bleeding.
Which one of the following management approaches is most appropriate regarding this patient's supratherapeutic international normalized ratio?
Answer
Hold the warfarin and administer 10 mg of vitamin K intravenouslyContinue the warfarin at a 25% lower doseHold the warfarin and transfuse 4 units of fresh frozen plasmaContinue the warfarin at the current doseHold the warfarin and administer 2.5 mg of vitamin K by mouth
The most appropriate management of a warfarin-treated patient who has an elevated international normalized ratio >10 without bleeding is to administer low-dose vitamin K by mouth and withhold warfarin.
Vitamin K is appropriate for the reversal of anticoagulation in asymptomatic patients with an elevated international normalized ratio (INR) who are at increased risk for bleeding, require surgery, or have serious bleeding. An INR greater than 4.0 has been associated with an increased risk for bleeding, and the risk for intracranial hemorrhage increases approximately twofold for every one-unit increase in INR.
The factors associated with an increased risk for bleeding are older age, alcohol use, and the presence of kidney failure.
Patients with an INR between 4.5 and 10 who do not have bleeding can be managed by withholding warfarin; reversal with vitamin K is not routinely recommended for these patients. If the INR is >10 with no evidence of bleeding (as in this case), warfarin should be withheld, and the use of oral vitamin K is suggested at a dose of 2.5 to 5 mg. Vitamin K will quickly lower the INR and allow warfarin therapy to be reinstituted at a lower dose. This strategy has not been shown to decrease bleeding events, but it is appropriate for patients considered to be at high bleeding risk.
An acceptable alternative strategy for managing a warfarin-treated patient who has an increased INR but a lower bleeding risk than this patient would be to simply withhold warfarin therapy for several doses without administering vitamin K.
When reinstituting therapy, it would be appropriate to either lower the dose of warfarin or replace warfarin with one of the newer oral anticoagulants.
Continuing warfarin at the current dose would place the patient at a greatly increased risk of bleeding.
Continuing warfarin at a 25% lower dose would not produce a sufficiently rapid decline in the INR to the target range.
Treating with fresh frozen plasma is not indicated, because it could exacerbate congestive heart failure and the patient is not actively bleeding.
High-dose vitamin K (such as 10 mg) would result in an overcorrection of the INR and difficulty reinstituting therapy with warfarin. Intravenous treatment with vitamin K is generally avoided because this route of administration is associated with hypersensitivity or anaphylaxis.
Last reviewed Feb 2018. Last modified Feb 2018.
Crowther MA et al. Oral vitamin K versus placebo to correct excessive anticoagulation in patients receiving warfarin: a randomized trial. Ann Intern Med 2009 Mar 5; 150:293. > View Abstract
Holbrook A et al. Evidence-based management of anticoagulant therapy: antithrombotic therapy and prevention of thrombosis, 9th ed: American College of Chest Physicians evidence-based clinical practice guidelines. Chest 2012 Feb; 141:e152S. > View Abstract
The most appropriate initial treatment of this rhythm in a stable patient, shown in Figure 30-2, is:
FIGURE 30-2.
The correct answer is C. You answered A.
The correct answer is "C." This ECG is showing atrial fibrillation with rapid ventricular response (A. fib with RVR). Note the absence of p-waves and the irregular rate and rhythm at about 150 to 180 bpm. It is most important to slow down the ventricular rate. In a stable patient, this can be done by using a medication that can affect the AV node, such as a calcium channel blocker (diltiazem) or β-blocker (metoprolol). Amiodarone is not the appropriate answer as it is indicated for cardioversion or maintenance of a cardioverted rhythm. Adenosine slows down conduction through the AV node and typically won't terminate atrial arrhythmias. Adenosine may slow down ventricular response, but this is limited to its half-life of about 10 seconds. IV Lidocaine is typically utilized for the treatment of ventricular arrhythmias (ventricular tachycardia or ventricular fibrillation) and has not role in the treatment of atrial fibrillation, which is an atrial-based arrhythmia. See question 2.3.5.
New treatment for atrial fibrillation
Atrial fibrillation is an abnormal heart rhythm originating in the atria (top chambers of the heart). Instead of the impulse traveling in an orderly fashion through the heart, many impulses begin and spread through the atria, causing a rapid and disorganized heartbeat.
At one time, atrial fibrillation was thought to be a harmless annoyance. However, now it is known that chronic atrial fibrillation is associated with heart failure, blood clots, a five to sevenfold increase in stroke, and increased mortality from heart disease.
Conventional therapy of atrial fibrillation includes medications, electrical cardioversion, ablation of the atrioventricular node (AV node) followed by a pacemaker, and various surgical procedures.
New medications: Tikosyn
Tikosyn (Dofetilide) is a new medication that is used to convert atrial fibrillation to normal sinus rhythm (NSR). Clinical trials have shown Tikosyn to be effective in converting patients to NSR in about 70 to 80 percent of cases. Tikosyn is an effective new drug, however there is risk of inducing more life-threatening arrhythmias when first beginning the drug and when changing doses. Therefore, Tikosyn is only available to hospitals whose physicians have received the proper training in initiation and dosing of the drug. Also, all patients who begin Tikosyn or have changes in dosing must be placed in the hospital for a minimum of three days for careful monitoring.
Improved safety during cardioversion: use of transesophageal echo:
Electrical cardioversion of patients with atrial fibrillation (AF) to normal sinus rhythm is frequently performed to relieve symptoms, improve cardiac performance and reduce the risk of stroke. If the patient has a preexisting thrombus (clot) inside the left atrial appendage of the heart, there is a risk of stroke during the procedure. In order to decrease this risk, patients undergoing electrical cardioversion are usually treated with anticoagulation medications (blood-thinners) for three weeks before and four weeks after the procedure. The trial studied the use of transesophageal echocardiography (TEE), with short-term anticoagulation, to lower the risk of stroke and bleeding for these patients. The study concluded that certain patients benefit from the TEE-guided strategy, allowing earlier cardioversion and less bleeding complications.
Surgical Procedures:
The Maze procedure (Cox-Maze procedure), developed by Dr. Jim Cox, began the surgical approach to treatment of atrial fibrillation. The surgery involves creating precise incisions in the right and left atria to interrupt the conduction of abnormal impulses and to direct normal sinus impulses to travel to the atrioventricular node (AV node) as they normally should. The Maze procedure has been very successful with a 95 % success rate. A number of surgeons have altered the traditional Cox-Maze procedure to focus mainly on the left atrium.
The success of the modified Maze procedures has supported the notion that the isolation of the pulmonary veins and portions of the left atrium can eliminate atrial fibrillation. This has encouraged surgeons to seek out other methods of isolation rather than cutting and sewing. Three alternative energy sources have been used surgically to treat atrial fibrillation: radiofrequency, microwave and cryothermy. The goal of all three is to produce lesions and ultimately scar tissue to block the abnormal electrical impulses from being conducted through the heart and promote the normal conduction of impulses through the proper pathway
Radiofrequency ablation uses radiofrequency energy to heat the tissue and produce lesions on the heart, eliminating the incisions necessary in the Maze procedure. A variety of surgical techniques related to the type of catheter used are common, the dose of energy, and the types of lesions created. Radiofrequency surgical ablation has proved to be successful in 80% of cases. The greatest risk of this procedure is damage to surrounding structures, such as the esophagus.
Cryothermy (also called cryoablation) uses very cold temperatures through a probe (called a cryoprobe) to create lesions. This technique is used commonly during arrhythmia surgery to replace the incisions made during the Cox-Maze procedure. This technique cures atrial fibrillation in close to 80% of people.
Microwave technology uses a special catheter (the Flex-4 catheter) to direct microwave energy to create several lesions on the heart. The lesions block the conduction of abnormal electrical beats and restore a normal heartbeat. The benefit of microwave radiation in comparison to other surgical ablation techniques, is that the depth and volume of heated tissue is more controlled and precise lesions are created, and less charring of the heart’s surface occurs, decreasing the risk of blood clots that may travel to the brain or other organs. Microwave energy cures atrial fibrillation in about 80% of people.
+++++++++++++++
Treatments for atrial fibrillation include medicines to control heart rate and reduce the risk of stroke, and procedures such as cardioversion to restore normal heart rhythm.
It may be possible for you to be treated by a GP, or you may be referred to a heart specialist (a cardiologist).
Some cardiologists, known as electrophysiologists, specialise in the management of abnormalities of heart rhythm.
You'll have a treatment plan and work closely with your healthcare team to decide the most suitable and appropriate treatment for you.
Factors that will be taken into consideration include:
The first step is to try to find the cause of the atrial fibrillation. If a cause can be identified, you may only need treatment for this.
For example, if you have an overactive thyroid gland (hyperthyroidism), medicine to treat it may also cure atrial fibrillation.
If no underlying cause can be found, the treatment options are:
You'll be promptly referred to your specialist treatment team if 1 type of treatment fails to control your symptoms of atrial fibrillation and more specialised management is needed.
Medicines called anti-arrhythmics can control atrial fibrillation by:
The choice of anti-arrhythmic medicine depends on the type of atrial fibrillation, any other medical conditions you have, side effects of the medicine chosen, and how well the atrial fibrillation responds.
Some people with atrial fibrillation may need more than 1 anti-arrhythmic medicine to control it.
A variety of medicines are available to restore normal heart rhythm, including:
An alternative medicine may be recommended if a particular medicine does not work or the side effects are troublesome.
Newer medicines are in development, but are not widely available yet.
The aim is to reduce the resting heart rate to under 90 beats per minute, although in some people the target is under 110 beats per minute.
A beta blocker, such as bisoprolol or atenolol, or a calcium channel blocker, such as verapamil or diltiazem, will be prescribed.
A medicine called digoxin may be added to help control the heart rate further.
Normally, only 1 medicine will be tried before catheter ablation is considered.
As with any medicine, anti-arrhythmics can cause side effects.
The most common side effects of anti-arrhythmics are:
Read the patient information leaflet that comes with the medicine for more details.
The way the heart beats in atrial fibrillation means there's a risk of blood clots forming in the heart chambers.
If these enter the bloodstream, they can cause a stroke.
Find out more about complications of atrial fibrillation
Your doctor will assess your risk and try to minimise your chance of having a stroke.
They'll consider your age and whether you have a history of any of the following:
You may be given medicine according to your risk of having a stroke.
Depending on your level of risk, you may be prescribed warfarin or a newer type of anticoagulant, such as dabigatran, rivaroxaban, apixaban or edoxaban.
If you're prescribed an anticoagulant, your risk of bleeding will be assessed both before you start the medication and while you're taking it.
Aspirin is not recommended to prevent strokes caused by atrial fibrillation.
People with atrial fibrillation who have a high or moderate risk of having a stroke are usually prescribed warfarin, unless there's a reason they cannot take it.
Warfarin is an anticoagulant, which means it stops the blood clotting.
There's an increased risk of bleeding in people who take warfarin, but this small risk is usually outweighed by the benefits of preventing a stroke.
It's important to take warfarin as directed by your doctor. If you're prescribed warfarin, you need to have regular blood tests and, after these, your dose may be changed.
Many medicines can interact with warfarin and cause serious problems, so check that any new medicines you're prescribed are safe to take with warfarin.
While taking warfarin, you should be careful about drinking too much alcohol regularly and avoid binge drinking.
Drinking cranberry juice and grapefruit juice can also interact with warfarin and is not recommended.
Rivaroxaban, dabigatran, apixaban and edoxaban are newer anticoagulants and an alternative to warfarin.
The National Institute for Health and Care Excellence (NICE) has approved these medicines for use in treating atrial fibrillation.
NICE also states that you should be offered a choice of anticoagulation and the opportunity to discuss the merits of each medicine.
Unlike warfarin, rivaroxaban, dabigatran, apixaban and edoxaban do not interact with other medicines and do not require regular blood tests.
In large trials, the medicines have been shown to be as effective or more effective than warfarin at preventing strokes and deaths. They also have a similar or lower rate of major bleeding.
You can read more about rivaroxaban, dabigatran and apixaban in the NICE guidance on managing atrial fibrillation.
Edoxaban is recommended as an option for preventing stroke, heart disease and coronary artery disease in people with atrial fibrillation who have 1 or more risk factors, such as:
You can read the NICE guidance about Edoxaban for preventing stroke and systemic embolism in people with non-valvular atrial fibrillation.
Cardioversion may be recommended for some people with atrial fibrillation.
It involves giving the heart a controlled electric shock to try to restore a normal rhythm.
Cardioversion is usually carried out in hospital so the heart can be carefully monitored.
If you have had atrial fibrillation for more than 2 days, cardioversion can increase the risk of a clot forming.
In this case, you'll be given an anticoagulant for 3 to 4 weeks before cardioversion, and for at least 4 weeks afterwards to minimise the chance of having a stroke.
In an emergency, pictures of the heart can be taken to check for blood clots, and cardioversion can be carried out without going on medication first.
Anticoagulation may be stopped if cardioversion is successful.
But you may need to continue taking anticoagulation after cardioversion if the risk of atrial fibrillation returning is high and you have an increased risk of having a stroke.
Catheter ablation is a procedure that very carefully destroys the diseased area of your heart and interrupts abnormal electrical circuits.
It's an option if medicine has not been effective or tolerated.
Catheters (thin, soft wires) are guided through 1 of your veins into your heart, where they record electrical activity.
When the source of the abnormality is found, an energy source, such as high-frequency radiowaves that generate heat, is transmitted through 1 of the catheters to destroy the tissue.
The procedure usually takes 2 to 3 hours, so it may be carried out under general anaesthetic, which means you're unconscious during the procedure.
You should make a quick recovery after having catheter ablation and be able to carry out most of your normal activities the next day.
But you should not lift anything heavy for 2 weeks, and driving should be avoided for the first 2 days.
A pacemaker is a small battery-operated device that's implanted in your chest, just below your collarbone.
It's usually used to stop your heart beating too slowly, but in atrial fibrillation it may be used to help your heart beat regularly.
Having a pacemaker fitted is usually a minor surgical procedure carried out under a local anaesthetic (the area being operated on is numbed and you're conscious during the procedure).
This treatment may be used when medicines are not effective or are unsuitable. This tends to be in people aged 80 or over.
Find out more about pacemaker implantation
Page last reviewed: 24 April 2018
Reference:https://www.nhs.uk/conditions/atrial-fibrillation/treatment/
AF can have adverse consequences related to a reduction in cardiac output and to atrial and atrial appendage thrombus formation.
++++++++++++++++++++
If no underlying cause can be found, the treatment options are:
You'll be promptly referred to your specialist treatment team if 1 type of treatment fails to control your symptoms of atrial fibrillation and more specialised management is needed.
Medicines called anti-arrhythmics can control atrial fibrillation by:
The choice of anti-arrhythmic medicine depends on the type of atrial fibrillation, any other medical conditions you have, side effects of the medicine chosen, and how well the atrial fibrillation responds.
Some people with atrial fibrillation may need more than 1 anti-arrhythmic medicine to control it.
A variety of medicines are available to restore normal heart rhythm, including:
An alternative medicine may be recommended if a particular medicine does not work or the side effects are troublesome.
Newer medicines are in development, but are not widely available yet.
The aim is to reduce the resting heart rate to under 90 beats per minute, although in some people the target is under 110 beats per minute.
A beta blocker, such as bisoprolol or atenolol, or a calcium channel blocker, such as verapamil or diltiazem, will be prescribed.
A medicine called digoxin may be added to help control the heart rate further.
Normally, only 1 medicine will be tried before catheter ablation is considered.
As with any medicine, anti-arrhythmics can cause side effects.
The most common side effects of anti-arrhythmics are:
Read the patient information leaflet that comes with the medicine for more details.
The way the heart beats in atrial fibrillation means there's a risk of blood clots forming in the heart chambers.
If these enter the bloodstream, they can cause a stroke.
Find out more about complications of atrial fibrillation
Your doctor will assess your risk and try to minimise your chance of having a stroke.
They'll consider your age and whether you have a history of any of the following:
You may be given medicine according to your risk of having a stroke.
Depending on your level of risk, you may be prescribed warfarin or a newer type of anticoagulant, such as dabigatran, rivaroxaban, apixaban or edoxaban.
If you're prescribed an anticoagulant, your risk of bleeding will be assessed both before you start the medication and while you're taking it.
Aspirin is not recommended to prevent strokes caused by atrial fibrillation.
People with atrial fibrillation who have a high or moderate risk of having a stroke are usually prescribed warfarin, unless there's a reason they cannot take it.
Warfarin is an anticoagulant, which means it stops the blood clotting.
There's an increased risk of bleeding in people who take warfarin, but this small risk is usually outweighed by the benefits of preventing a stroke.
It's important to take warfarin as directed by your doctor. If you're prescribed warfarin, you need to have regular blood tests and, after these, your dose may be changed.
Many medicines can interact with warfarin and cause serious problems, so check that any new medicines you're prescribed are safe to take with warfarin.
While taking warfarin, you should be careful about drinking too much alcohol regularly and avoid binge drinking.
Drinking cranberry juice and grapefruit juice can also interact with warfarin and is not recommended.
Rivaroxaban, dabigatran, apixaban and edoxaban are newer anticoagulants and an alternative to warfarin.
The National Institute for Health and Care Excellence (NICE) has approved these medicines for use in treating atrial fibrillation.
NICE also states that you should be offered a choice of anticoagulation and the opportunity to discuss the merits of each medicine.
Unlike warfarin, rivaroxaban, dabigatran, apixaban and edoxaban do not interact with other medicines and do not require regular blood tests.
In large trials, the medicines have been shown to be as effective or more effective than warfarin at preventing strokes and deaths. They also have a similar or lower rate of major bleeding.
You can read more about rivaroxaban, dabigatran and apixaban in the NICE guidance on managing atrial fibrillation.
Edoxaban is recommended as an option for preventing stroke, heart disease and coronary artery disease in people with atrial fibrillation who have 1 or more risk factors, such as:
You can read the NICE guidance about Edoxaban for preventing stroke and systemic embolism in people with non-valvular atrial fibrillation.
Cardioversion may be recommended for some people with atrial fibrillation.
It involves giving the heart a controlled electric shock to try to restore a normal rhythm.
Cardioversion is usually carried out in hospital so the heart can be carefully monitored.
If you have had atrial fibrillation for more than 2 days, cardioversion can increase the risk of a clot forming.
In this case, you'll be given an anticoagulant for 3 to 4 weeks before cardioversion, and for at least 4 weeks afterwards to minimise the chance of having a stroke.
In an emergency, pictures of the heart can be taken to check for blood clots, and cardioversion can be carried out without going on medication first.
Anticoagulation may be stopped if cardioversion is successful.
But you may need to continue taking anticoagulation after cardioversion if the risk of atrial fibrillation returning is high and you have an increased risk of having a stroke.
Catheter ablation is a procedure that very carefully destroys the diseased area of your heart and interrupts abnormal electrical circuits.
It's an option if medicine has not been effective or tolerated.
Catheters (thin, soft wires) are guided through 1 of your veins into your heart, where they record electrical activity.
When the source of the abnormality is found, an energy source, such as high-frequency radiowaves that generate heat, is transmitted through 1 of the catheters to destroy the tissue.
The procedure usually takes 2 to 3 hours, so it may be carried out under general anaesthetic, which means you're unconscious during the procedure.
You should make a quick recovery after having catheter ablation and be able to carry out most of your normal activities the next day.
But you should not lift anything heavy for 2 weeks, and driving should be avoided for the first 2 days.
A pacemaker is a small battery-operated device that's implanted in your chest, just below your collarbone.
It's usually used to stop your heart beating too slowly, but in atrial fibrillation it may be used to help your heart beat regularly.
Having a pacemaker fitted is usually a minor surgical procedure carried out under a local anaesthetic (the area being operated on is numbed and you're conscious during the procedure).
This treatment may be used when medicines are not effective or are unsuitable. This tends to be in people aged 80 or over.
++++++++++++++++++
Anticoagulation with warfarin has significant risks, so should be reserved for patients with high CHADS2 scores.
Mortality outcomes are similar for patients with atrial fibrillation who are treated with rhythm control compared with those treated with rate control.
+++++++++++++++
Antithrombotic therapy in both the acute and the chronic management of AF.
Be familiar with the various options for anticoagulation, including the newer antithrombin and anti-Xa agents.
A 75-year-old man is found to have asymptomatic AF. Which of the following is the most common complication of his AF long term?
The correct answer is B.
B. The two complications associated with AF are stroke and cardiomyopathy. Stroke is two to three times more likely in patients with AF than in the general population. While cardiomyopathy is also a complication of AF, it is far less common than thromboembolism.
Content 2
Content 3